You are on page 1of 59

WILLS TRUSTS AND ESTATES

DUKEMINIER & STIKOFF WILLS, TRUSTS, & ESTATES (9TH EDITION)


Professor Puckett, Penn State Law.
Intestacy
will
intestate Governed by the default pg 63-81, 81-91
testate -- when someone dies with a
rules of intestacy, when someone dies without a will.
Primary purpose of intestacy statute if to carry out the probable intent of the
typical intestate decedent default rules for property succession at death
UPC 2-101 Intestate Estate
2-102 Share of Spouse
2-103 Share of heirs other than surviving spouse
S= surviving spouse; D = surviving descendant(s) of decedent; P = surviving parent(s) of decedent;
BorS = surviving siblings of decedent, surviving descendants of decedents parents; G = surviving grandparent
of decedent; GD surviving descendants of grandparents
Facts
S; no D; no P
S; D

UPC Authority
2-102(1)(A)
2-102(1)(B)
2-102(3)

2-102(4)
S; no D; P

2-102(2)

No S; D

2-103(a)(1)

No S; No D; P
No s; no D; no P; BorS

2-103(a)(2)
2-103(a)(3)

No s; no D; no P; NO borS; G
or GD

2-103(a)(4)

2-103(a)(5)

No s; no D; no P; no BorS;

2-103(b)

Disposition
All to s
All to S if all D are also Ds
and S has no other surviving
descendants
$225,000 plus half of the
rest to S if D are also Ss
and S has other
descendants; other half to D
$1500,000 plus half of the
rest to S if one or more D is
not Ss; other half to D
$300,000 plus of the rest
to S ; other one-fourth the P
All to D (per capita at each
generation)
All to P
All to BorS (per capita at
each generation)
If both paternal and
maternal G or GD, one half
to paternal G or GD and one
half to maternal G or GD (all
to G or if non, per capita at
each GD generation)
If survivors on one side
only, all to G or GD on that
side (all to G or, if none, per
capita at each GD
Generation
Stepchildren or, if none then

no g or gd

2-105

Escheat to state; therefore,


no laughing heirs

Heir apparent people that have a mere expectancy that is both contingent on their
surviving A and defeasible by As contrary dispotion by will, will substitute or lifetime gift
Devisee; legatee; beneficiary a person named in a will.
Estate Plan By Default
Summary of UPC Intestacy Provisions
Basic Structure of Intestate Succession
Domestic Partners and Same Sex Marriage policies that underpin the spousal intestate
share seem to also apply to long-term cohabitating partners.
The Problem of simultaneous death A person succeeds to the property of a decedent
ONLY if the person survives the decedent for an instant time.
Arises in wills, trusts and other modes of nonprobate transfer in which the
governing instrument does not avoid the problem by requiring a beneficiary to
survive the donor by a stated period of time.
Uniform Simultaneous Death Act provided that if there is no sufficient
evidence of the order of deaths each was deemed to have predeceased the other,
so neither inherited from the other.
o Issue of what is sufficient evidence, remedy = UPC 2-104 and 2-702
2-104 and 2-702 provides that an heir, devisee or life insurance
beneficiary who fails to survive by 120 hours (5 days) is deemed to
have predeceased the decedent. under the amended UPC claimant
must establish survivorship by 120 hours by clear and convincing
evidence.
Janus v. Tarasewicz
Facts: Husband and wife died after taking cyanide laced Tylenol (on accident) .
Wife was on life support for two days while hubby died upon arrival at the hospital.
Issue of who died first
Holding: Court found sufficient evidence that she outlived her husband and
appellate affirmed.
Descendants
Representation
If a child dies before decedent and the child leaves descendants, all state provide
that the childs descendants shall represent the dead child and divide the childs
share among themselves.
Fundamental issue is whether the division into shares should begin at the
generational level immediately below the decent or at the closest generational
level with a descendant of the decedent alive.

English Per Stirpes / strict per stirpes


Followed by 1/3 of states.
Treats each line of descent equally
o The property is divided into as many shares as there are living children of the
designated person and deceased children who have descendants living.
o Children of each of the deceased descendant represent their deceased
parent and are moved into their parents position beginning at the first
generation below the designated person.
o Assures vertical equality at the expense of horizontal equality.
Modern Per Stirpes /per capita with representation
Less than states follow
Look first to see whether any children survived decedent
o Yes distribution is identical to English Per stirpes
o No estate is divided equally (per capita) at the first generation in which
there are living takers
Per Capita at Each Generation ( 1990 UPC)
Initial division of shares is made at the closest generation in which one or more
descendants are alive (as under modern per stirpes) HOWEVER the shares of the
deceased persons on that level are treated as one pot and are dropped down and
divided EQUALLY among the representatives in the next generation.

Representations in Wills and trusts


Which one rules depends on the state
Ancestors, Collaterals and Others

Parents no descendants? Goes to parents.


Other ancestors and collaterals no parents OR descendants? Goes to more remote
ancestors or collateral kindred. All persons who are related by blood to the decedent but
who are not descendants or ancestors are called collateral kindred.

TWO
BASIC
SCHEMES
Parentelic system intestate estate passes to grandparents and their descendants
and if none to great-grandparents and their descendants and if none to great great
grandparents and so on down the line (parentila) descended from an ancestor until
an heir is found
Degree of relationship system intestate passes to the closest of kin, counting
degrees of kinship. (Table of consanguinity)
Laughing Heirs distant relatives
Stepchildren and In laws 1/3 of states and the UPC recognize stepchildren as potential
heirs.
Half Bloods majority of states and the UPC relatives of a half blood is treated the same
as a relative of a whole blood
Escheat if intestate decedent leaves NO survivors entitled to take under the intestacy
statute her probate property escheats to the state under UPC 2-105.
Disinheritance by negative Will

UPC 2-101(b) authorizes a negative will by way of an express disinheritance


provision. The barred heir is treated as if he disclaimed his intestate share which
means he is treated as having predeceased the decedent.

Bars to inheritance: Advancements 126-129; Slayer Rule 132-139; Disclaimer 140-145


Advancement and Hotchpotch
If a child wishes to share in the intestate distribution of a deceased parents estate, the
child must permit the administrator to include in the determination of the distributive
shares the value of any property that the decedent, while living gave the child by way of
an advancement
Advancement at common law any lifetime gift by the decedent to a child was
presumed to be an advancement; prepayment of the childs intestate share.
o To avoid application of the doctrine, the child had the burden of est. that the
transfer was intended as an absolute gift that was not to be counted against
the childs share of the estate.
o When a parent makes an advancement to a child and the child
PREDECEASES the parent, the amount of the advancement is deducted from
the shares of the childs descendants if other children of the parent survive.
Hotchpot if a gift is treated as an advancement it is account for in distributing
the decedents estate by bringing into hotchpot
o O dies with no spouse but children A, B and C with an estate worth 50k.
A received advancement of 10k (to calculate shares for A, B and C you
ADD the 10k gift to the 50k estate.)
10k advancement + 50k estate = 60k hotchpot
60k divided by 3 descendants = 20k/each
A only gets 10k because she already received 10k

*** IF A was given 40k gift she would just be left out of the hotchpot because we know O
would want A to have at least 40k ***

It is not uncommon in wills and trusts to find a provision instructing that certain
lifetime gifts or distributions under the instrument are to be taken into account
when calculating later shares

Advancements in modern law Many states have reversed the common law a lifetime
gift is presumed NOT to be an advancement unless it is shown to have been intended as
such
Uniform Probate Code 1990 2-109
Inter vivos gifts do not constitute an advancement unless a writing indicates that
the donor intended the gift to constitute an advancement
o Writing requirement (a) if the donor creates the writing, the writing must be
made contemporaneously with the inter vivos gift (b) if the donee creates
the writing, the writing may be made at any time
o Donee predeceases unlike commo law approach if the donee predeceases
the donor the inter vivos gift to the donee qualifies as an advancement. The
advancement does not count against the share of the donors estate going to
the donees issues unless the writing expressly provides so
Guardianship and Conservatorship of Minors 129
Guardian of the person
Guardian has responsibility for the minor childs custody and care.
o If both parents die while the child is a minor the court will appoint a guardian
of the person usually from among the nearest relatives.
o A guardianship of the person terminates when the minor reaches the age of
majority, dies or is adopted.
o Covered in UPC 5-201 to 5-210
o NO AUTHORITY to deal with the childs property
Property management options
(1)Guardianship of the property
o Has the duty to preserve the specific property left to the minor and deliver it
at the age of 18, unless the court approves a sale, lease or mortgage. The
guardian can only use income from the property to support the ward.
(2)Conservatorship
o Only one trip to the court house annually fro accounting; more flexible and
expires when the minor reaches the age of majority
o Conservator is given title as trustee to the protected persons property
along with investment powers similar to those of a trustee.
(3)Custodianship
o Custodian is a person who is given property to hold for the benefit of a
minor under the uniform transfers to minors act.
o Custodian has right to manage the property and to reinvest it however the
custodian is a fiduciary and is subject to the :standard care that would be
observed by a prudent personal dealing with property of another
(4)Trusteeship
o Available only to persons who create them during life inter vivos trust OR
who die testate and create one by will a testamentary trust
o Most flexible. Can tailor trust specifically to circumstances and desires. A
trust can postpone possession.

Bars to Succession
o
o

Rule that prohibits a slayer from inheriting from his victim

Voluntary disclaimer
The Slayer Rule
In re Estate of Mahoney
Facts: Mahoney died intestate of gunshot wounds, wife convicted of
manslaughter. Survived by wife, father and mother, father was administrator.
Issue: Can a widow convicted on manslaughter in connection with the death of her
husband inherit from his estate
Holding: probate court cannot impost a constructive trust this needs to be done
by the Court of chancery. Reverse and remanded to the court of chancery.
Notes:
o Courts in states without slayer statutes have 3 options
1. Legal title passed to the slayer may be retained in spite of the crime
(no inheritance = additional punishment and you dont want to punish
someone twice for the same crime)
2. Legal title will not pass no one should profit from wrongdoing
3. Legal title passes, but equity holds him as a constructive trustee for
the next heirs
Constructive trust:
Mercy Killing in In Re Estate of Schunk W provided H with the loaded shotgun that H
used to kill himself. The court held that Ws assisting H to commit suicide was not an
unlawful and intentional killing within the meaning of the Wisconsin slayer statute.
Disclaimer
o Disclaimer is a refusal to take the property
From common law to statutory law
At common law if an heir renounces, the law treated it as if the title had passed to
the heir, to the next intestate success. Almost all states have chosen to treat it as
if the disclaimer died before the distribution
Avoiding Taxes Most state disclaimer statutes require that a disclaimer be made within
9 months of the creation of the interest being disclaimed UPC 2-1101 to 2-1117
Avoiding Creditors most disclaimer statutes provide that a disclaimer relates back for
all purposes to the date of the decedents death UPC 2-1106. In an intestate estate the
disclaimer takes effect as of the time of the intestates death UPC 2-1106(b)(1)
SCOTUS held that you cannot disclaim in order to avoid creditors. You still inherited
the property (or you wouldnt have been able to pass it off) so you are subject to
liens

Will formalities and strict compliance 147-165; Purging Statutes 167 SKIM
model execution 167-171

Willis: Formalities and Forms


Execution of Wills
Worst evidence problem in discerning the authenticity, the voluntariness and the
meaning of a will.
Wills act deals with Attested Wills, holographic wills and notarized wills
Attested Wills
3 Core Formalities
o writing
o signatures
o attestation
The Functions of Formalities
o Evidentiary function supply satisfactory evidence to the court
o Ritual function (cautionary function) a ceremony supposedly impresses
upon the transferor the significance of the event
o Protective function purpose of safeguarding the testator
o Channeling function standardization of form simplifies administration

UPC 2-502
Substantial Compliance with the Wills Act want people to conform with wills act so
they dont have to sit around wondering if something was meant to be a will or not.
Strict Compliance Rule
o Under traditional law for a will to be admitted to probate it must be in strict
compliance with the formal requirements of the applicable Wills Act.
o Must be in writing, signed by the testator, and then attested by two
witnesses
o Guards against a spurious funding of authenticity a false positive
o Est. a conclusive presumption of invalidity for an imperfectly execute instrument
the strict compliance rule denies probate even if the defect is innocuous and there
is overwhelming evidence of authenticity false negative
In re Groffman
Facts: Groffman died 3 yrs after executing a will in the home of his friends the
blocks. The will had been prepared by a lawyer who gave it to Groffman to execute
on his own. Under the will the daughter/step daughter would take home. Widow
contested the will. Litigation focused on whether the witnesses were present
together when he signed. The two witnesses were not present at the same time
when signing will (one was in living room one was in dining room)
Holding: Witnesses must be present at the same time for a will be to considered
valid
Stevens v. i've briefed 15 cases to orph
Facts: Miller when to Bank to execute his will. Bank employee/public notary, saw
him execute the will. She then took the will to be signed by 2 other bank
employees who didnt see him execute the will. Challenge was brought by nieces.
Statute: the signature shall be made or the will acknowledged by him in the
presence of at least two competent witnesses present at the same time, and such
witnesses shall subscribe the will in the presence of the testator and of each other
but no form of attestation shall be necessary
Holding: Will was not properly executed. This is an example of strict compliance

Notes:

The Meaning of Presence


Line of sight the testator does not actually have to see the witness sign but must be
able to see them were the testator to look. (exception is made for a blind testator)
Conscious Presence the witness is in the presence of the testator if the testator,
through sight, hearing or general consciousness of events, comprends that the witness is
in the act of signing
o Test of mental comprehension
Uniform Probate Code dispenses altogether with the requirements that the
witnesses sign in the testators presence. 2-502(1) page 149
Witnesses most jx rq the testator sign or acknowledge in the presence of at least two
witnesses who are present at the same time. The witness must sign the will and in some
jx the witnesses must know that what they are signing is the testator will
UPC does not require the witnesses to sign the will in either the testators presence
or the presence of each other UPC 2-502(a)(2) Many traditional wills act statutes
require the witness to sign the will in each others present
The Signature Requirement
o All states including the UPC requires the testator to sign the will
o Purpose of signature rq is to provide evidence of finality, distinguishing a will from
mere drafts or notes and to provide evidence of genuineness.
Signature by Mark, with Assistance, or by Another
o Full signature is preferable, a mark, cross, abbreviation or nickname can be
sufficient.
Order of signing IN GENERAL the testator must sign or acknowledge the will before
the witnesses attest. HOWEVER if they all sign as part of a single or continuous
transaction, the exact order of signing is not critical.
Subscription and Addition After Signature some states have adopted the English
wills act rq that a will be signed at the foot or end thereof this is called subscription
Delayed Attestation under UPC 2-502(a)(3)(A) witnesses must sign within a
reasonable time which would extend until after the testators death.
The meaning of writing and video or electronic Wills a will need not be on
paper, all that is required is a reasonably permanent record of the markings that make
up the will
Video Wills A dvd/video tape does not comply with the requirement that the will be a
signed writing In Re Estate of Reed.
Electronic Wills An electronic will probably does not satisfy the writing (or signature)
requirements of an ordinary Wills act, but such a will might be allowed under the
substantial compliance doctrine or the harmless error rule.
Purging statute Allows a will attested by an interested witness to be admitted to
probate but it voided any bequest to the interest witness. W will attested by an
interested witness would be valid but the witness would not take his devise
Interested Witness under the UPC 167 not many states follow, do not require that
any of the witnesses be disinterested UPC 2-505(b) provides that a will is valid even if
witnessed by an interested person and without purging the interested witness of his
devise.

Relief from Strict Compliance 179-197


Substantial Compliance Doctrine
o Key question is whether the manner in which an instrument was executed satisfied
the purposes of the Wills Act formalities. IF SO instrument should be deemed in
substantial compliance with the Wills Act and admitted to probate.
Harmless Error Rule
o The court may excuse noncompliance if there is a clear and convincing evidence
that the decedent intended the document to be his will
o
Uniform Probate Code 2-503

In re Estate of Hall
Facts: Joint will to represent the desires of couple. Signed and notarized but there
were not attesting witnesses. It follows UPC code section 2.3 that allows for
harmless error if they can show that, by clear and convincing evidence the
document is intended to be the will. Pretty clear what the donor intent was, court
used harmless error to solve the defect in attestation.
o Clear and convincing evidence
o Writing, signature and attestation
In re Probate of Will and Codicil of Macool
Facts: Prior to husbands death, wife executed will and codicil but later there were
notes in addition. She went to see her lawyer, who dictated her will entirely in her
notes. Dictation was typed and the word DRAFT: placed on the document. There
was evidence that the wife sat through the dictation, went to lunch, was supposed
to sign it later that day. She died without signing
Issue: Can you use harmless error?

Holding: Must show by clear and convincing evidence this was the donors intent
the process here is NOT cured Harmless error = high level of proof potentially
more than just intent, must have shown they reviewed the document in question.
o Probating an Unsigned Instrument harmless error rule may be invoked to
probate an unsigned instrument other than in a switched wills case BUT the
court requires that the proponent to prove by clear and convincing evidence
that the decedent in fact reviewed the instrument and thereafter assented to
it.
o Casual Writings

Notarized Wills
o UPC 2-502(a)(3)
o Validates a will that has been acknowledge by the testator before a notary
public or other individual authorized by law to take acknowledgements)
o Provides a will is valid if it is signed by two witnesses or if it is notarized.
Holographic Wills 197-215
Holographic Wills written by the testators hand and signed by the testator. Need not
be attested by witnesses.
o PROS & CONS: They are indispensible for testators who are either unwilling or
unable to commission a traditional will. The authors of holographic wills are not
foolish or unreliable. Some argue they breed litigation and are inartful.
o
Discerning testamentary Intent
In re Kimmels Estate
Facts: A letter was mailed by decedent to two of his children George and Irvin who
were named beneficiaries. Addressed to their resident and he died on the
afternoon of the same day.
Issue: Is the paper testamentary in character? Is the signature in compliance with
the Wills Act? (used the word father instead of name)
Holding: The words of if enny thing happens support testamentary intent. This
is not an ordinary will BUT his intent to execute is apparent.
Notes:
o Testamentary intent?
o The pros and cons of holographic wills
o Conditional Wills
Preprinted will forms
o If signing a preprinted will form and you complete it by hand and then sign it but it
does not have it attested by two witnesses, if strict compliance with the wills act is
required the form is not entitled to probate as a formal will
o However, it might be entitled to probate as a holographic will if enough of its
text was handwritten by the decedent
In re Estate of Gonzalez
Facts: Gonzalez visited his bro Joe and Joes wife. Gonzalez was planning to fly to
FL and he wanted to prepare his will before leaving. He showed Elizabeth and
Joseph two copies of a preprinted will form. On the first copy, he handwrote it. She

testified that he had already filled out the form before she saw it and didnt see
him sign it Gonzalez gave a copy of the blank form to Joe/Elizabeth and it was
signed but not filled out. Three of his children tried to probate the will; other
children filed a summary judgment motion.
Holding: Valid holographic will. Printed portions of a will can be incorporated into
a holographic will where the court needs testamentary intent, considering all of the
evidence in the case. UPC allows for them.
o Incorporation by reference and surplusage
o Harmless error and preprinted Forms

Signature and Handwriting


Signature almost all states allow the signatures to be at the end, at the beginning, or
anywhere else. If it is not signed at the end, there may be doubt if the signature was
intended. Implies that you read the whole will.
The extent of the testators handwriting
- First generation: Entirely written , signed and dated
- Second generation (1969 UPC) material provisions
- 3rd generation (1990 UPC) material portions and extrinsic evidence
Signature and Handwriting
A holographic will must be
o Written by the testators hand and
o Signed by the testator
Signature
o In almost all states permitting holographs, the will may be signed at the end,
at the beginning or anywhere else on the face of document
o But, if not signed at the end, there may be doubt about whether the
decedent intended his name to be a signature.
Williams v. Towle The testator did not sign his name at the end, but he did write his
name in block letters on the top of the first page The court admitted the will to probate.
Extrinsic Evidence
In re Estate of Kuralt
Facts: Charles Kuralt (tv actor) had 20 yr relationship with Shannon. Both wanted
to keep it a secret. In 1989 he executed holographic will that gave all of his interest
in a particular property to Shannon. Then in 1994, he executed a formal will that
didnt mention any of the property he owned. He intended to transfer a 90 acre
parcel to Shannon in 1997 but became sick. Upon going into the hospital, he wrote
her a letter about his intent along with $17k. She sought to probate the letter as a
holographic codicil estate objected, trial court found in her favor
Holding: This is a valid holographic codicil. There was extrinsic evidence of his
intent and he even used the word inherit for the specific bequest
Notes:
o Death and taxes
o Testamentary intent or intended disposition
Revocation of Wills 215-229; 239-241; Revival of Wills 229-234, 238-239
Revocation of Wills

o An important corollary to the principle of freedom of disposition is that a person


remains free to rework her estate plan until the moment of death. A will is
ambulatory subject to modification or revocation by the testator at any time prior
to death.
Revocation by Writing or by Physical Act
o All states permit revocation of a will:
o 1. By a subsequent writing executed with Wills act formalities AND
o 2. By a physical act such as destroying, obliterating, or burning the will.
o An oral declaration that the will is revoked, without more, is not enough to revoke
the will.
o If a duly executed will is NOT revoked with accordance with applicable statute, the
will must be admitted to probate UPC 2-507
Express and Implied Revocatory Writing
o Express revocation
o Writing executed with Wills Act formalities may revoke an earlier will in
WHOLE or PART by express revocation
o Most wills open with an express revocation clause
John Doe, a resident of ______,______ make this my will and revoke all
prior wills and codicils
o Implied revocation
o May revoke in whole or part by inconsistency (implied revocation)
o Issue arises when a testator executes a subsequent will that does not include
an express revocation clause. Question is whether the testator intended the
subsequent will to replace a prior will in whole or in part, or if instead he
intended the subsequent will to supplement the prior will.
o Modern View
o UPC
o To treat a subsequent will that does not expressly revoke a prior will BUT
makes a complete disposition of the testators estate, as presumptively
revoking the prior will by inconsistency 2-507(c)
IF the subsequent will DOES NOT make a complete disposition of the
testators estate it is viewed as a codicil
Codicil a testamentary instrument (i.e. will) that supplements rather
than replaces an earlier will; the later codicil supersedes the earlier will
to the extent of inconsistency between them
Formalities, writing and physical acts
Thompson v. Royall
Facts: Mrs. Kroll properly executed her will, then wanted to destroy and revoke the
will. One of her attorneys recommended keeping a copy of her old will just in case
she wanted to make a new one. She signed on the old will that it was null and
void and was only being kept for memorandum on the back of the cover sheet of
the will. The will was then entered into probate.
Issue: Did she properly revoke her will?
Holding: If written words are used for the purpose of revoking a will, they cannot
merely be on a blank part of the paper. The writing must mutilate, erase, deface or
come into contact with the written part of the will
o UPC 2-507(a)(2) changes this allows for revocation by a physical act of
cancellation whether or not the cancellation touches any words.

Notes:
o Intent to Revoke
o Malpractice
o Cancellation and Harmless Error in a revocatory Writing

In Re Estate of Stoker
Facts: Stoker drafted a will, and he intended for his then gf Gularte to be the
executer of the will. He died in 2008 and Gularte tried to introduce the will into
probate. Stokers daughter objected more recent will/they broke up. Daughter
brought in the will and a witness saying it was the one Stoker wanted. Holographic
will , written by the witness but Stoker peed on a copy of the old will an burned it
Holding: A will may be revoked where the testator executes a subsequent
inconsistent will or where he or she burns or destroys the will.
Notes:
o Harmless error and revocation by writing
o Revocation by physical act
o Revocation by physical act on a copy
Presumption of Physical Act Revocation
Harrison v. Bird
Facts: Daisy Speer executed a will and named Katherine Harrison the beneficiary.
The original was retained by Speers atty and Harrison had a duplicate. Mrs. Speer
later called her atty and said she wanted to revoke her will and atty/secretary tore
up the will. The atty then mailed the pieces and a note to Speer to verify it was
destroyed and told her she was without a will. She died and Harrison tried to
probate the duplicate. Circuit court 1) the will wasnt lawfully revoked wasnt
done in her presence 2) no ratification of the destruction of her will 3) couldnt find
the destroyed pieces of her will so presumption that she revoked it herself
Holding: Court held that speer destroyed her will/revoked it. The burden is on
Harrison to present evidence to rebut that.
Notes: Court relied instead on the PRESUMPTION of revocation that arises when a
will last known to be in the testators possession cannot be found (or is found in
mutilated condition) the law presumes that the will cannot be found because the
testator destroyed or mutilated it with the intent to revoke it.
Lost wills and the Presumption of Revocation assumption of revocation if the will
in the testators possession cannot be found OR found mutilated. Burden of proof on the
proponent of the will newer court requires a preponderance of the evidence
1. If you use Will B to revoke Will A, then destroy Will B, Will A will not be reinstated
2. Partial revocation UPC 2-507 authorized partial revocation by a physical act
3. Dependent Relative Revocation if a testator undertakes to revoke his will
upon a mistaken assumption of law or fact, under the doctrine of dependent
relative revocation (DRR) the revocation is ineffective if the testator would not
have revoked the will but for the mistaken belief.
o A partial or complete revocation is ineffective if made:
o 1. In connection with an attempted to achieve a dispositive objective that
fails under law
o 2. If a testator undertakes to revoke his will upon a mistaken assumption
of law or fact, the revocation is ineffective if the testator wouldnt have

revoked the will but for the mistaken belief. The basis for disregarding the
revocation is the testators mistaken belief.
(a) the former will is revived when the new will from which the
former will was revoked is found to be invalid
(b) the presumption established in subsection (a) is allowing the
revocation to remain in effect would be more consistent with the
testators probable intention.
Partial Revocation by Physical Act most states authorize partial revocation by
physical act.
Dependent Relative Revocation
LaCroix v. Senecal
Facts: Testator a will, then wrote a codicil. Original language: bequeath to my
nephew Nelson Lamoth; new language added Marcisse Lamoth aka Nelson
Lamoth. The codicil was signed by her husband, a named beneficiary.
Holding: The codicil was just an unnecessary correction and failed under
applicable law. The codicil should NOT be treated as making the original will
revoked or ineffective.
Notes: If a testator cancels or destroys a will with the intention of making a new
one and the new will is not made or fails it will be presumed the testator preferred
the old will to intestacy and the old will be admitted to probate in the absence of
evidence overcoming this presumption Revocation is not effective where the
intentions to revoke are conditional and the conditions are not fulfilled.
Limitations on DDR Courts have held that DRR applies ONLY:
1. if there is an alternative plan of disposition that fails OR
2. if the mistake is recited in the terms of the revoking instrument or possible is
established by clear and convincing evidence.
Revival of Revoked Wills
In re Estate of Alburn
Facts: Ottilie Alburn died; her sister Adele filed a petition for appointment as
administrator of the estate which alleged that she died intestate. Viola Henkey a
grand niece, filed a petition for the probate of a will which Alburn executed in
Milwaukee, in which she was named a legatee and executrix. Lulu Alburn and Doris
filed another petition of a will from Kankakee. The county court held a hearing on
the petitions Kankakee Will had been destroyed under the belief that the
Milwaukee will could be revived. Court probated the Kankakee Will Adele
appealed. The wills named different beneficiaries.
Issue: Did the deceased revoke the Kankakee will under the mistaken belief that
she was reinstating the Milwaukee will by clear preponderance of the evidence?
Holding: The testator desired her first will to stand even though she revoked it
making another will. No evidence that she wanted to die intestate. You can use
DRR to try to argue either way, depending on your state. Many states would reject
reviving the Kankakee.
Notes:
o UPC 2-508(a) if a subsequent will wholly revoked the previous will is itself
revoked by a physical act, the presumption is that the previous will remained
revoked

o UPC 2-509(b) if a subsequent will partly revoked the previous will is


revoked, the presumption is that the previous will is revived.
o UPC 2-509(c) if Will 2 is revoked by will 3, Will 3 does not revive will 1
unless the text of Will 3 indicates the result was intended.
Revocation by Operation of Law
Divorce In most states, statutes provide that a divorce revokes provisions for the
divorced spouse. In the remaining states, revocation occurs only if the divorce is
accompanied by a property settlement
UPC 2-804
Marriage in most states, a premarital will remains valid in spite of marriage. A
surviving pretermitted spouse is entitled to an intestate share of the estate, unless the
omission was intentional
Birth of children almost all states have pretermitted children statutes which give a
child born after the execution of a will a share in the parents estate.

Components of a Will 241-256


Components of a will
Doctrine of Integration all papers that are present at the time of execution and are
intended to be part of the will are treated as such. Pages should be stapled/fastened or
show a connection of language
In re Estate of Rigsby
Facts: Appeal of Betty Dorsey, sister of decedent JEassline from an order
admitting a holographic will to probate. The document had two pages of the will.
The error on appeal is a failure of the trial court to allow the second page to be
admitted as the page was not fastened. Both pages are handwritten and
initialed/dated on top. Only the first page is signed on the bottom and has
confirmed language for a list of items on the second page. The first page makes no
mention of page 2 and page 2 conflicts with page 1 about jewelry
Holding: The trial court did not err that page 1 was the entire holographic will
did not clearly appear that page 2 was intended to be included with the first.

Republication by Codicil
Publication of a will occurs when a testator conveys to the witnesses, by words or by
action that a document is the testators will.
Republication by Codicil a validly executed will is treated as re-executed
(republished) as of the date of the codicil.
o A will is treated as if it were executed when its most recent codicil was executed
whether or not the codicil expressly republishes the prior will unless the effect of so
treating it would be inconsistent with the testators intent.
Incorporation by Reference allows for a writing that was in existence but not
present at the time of execution that was not itself execute with testamentary formalities
to be absorbed into the testators wills.
UPC 2-510

Clark v. Greenhalge
Issue: Did a probate judge correctly conclude that a specific bequest of personal
property contained in a notebook were incorporated by reference into the will?
Holding: Yes. A notebook that gives guidance in distributing the testators estate
may be incorporated by reference to a will that includes language on how to
distribute. Doesnt matter that in the will it was called a memorandum but not on
the notebook.
Notes:
Johnson v. Johnson T a lawyer prepared 3 typewritten paragraphs
stating that the document was Ts will and making various bequests.
Typewritten text was not signed by T or witness and it appears to cut
off mid-sentence. Beneath the typewritten text at the bottom of the
page T wrote the following by hand To my brother James I give 10
dollars only. This will shall be complete unless hereafter altered,
changed or rewritten T signed and dated the document below the
handwritten portion. The court held that the valid holographic codicil
incorporated the prior will by reference and republished and validated
the prior will as of the state of the codicil thus giving effect to the
intention of the testator.
Berry v. Trible after lawyer sent T draft will, T made handwritten
changes to it signing on the bottom of each page. One of the pages T
wrote I give and bequeath all with an arrow pointing to an intended
beneficiary. Court held that the document could not be probated as a
holograph because the handwriting and typewritten text were
interwoven both physically and in sequence of thought
o Incorporation Not Recognized in NY, CT or LA
Subsequent Writings and Tangible Personal Property 2-513 , allows a testator
to dispose of tangible personal property by a separate writing, even if prepared after the
execution of the testators will provided that the will make reference to the separate
writing.
- the UPC allows the testator to reserve the power to make and then continue
revising a list of bequests of tangible property with out additional testamentary
formalities.
Acts of independent significance sometimes permits extrinsic evidence to identify
the beneficiaries or property passing under a will.
- If the beneficiary or property designations are identified by reference to acts or
events that have a lifetime motive and significance apart from their effect on the
will and the gift will be upheld under the doctrine of acts of independent
significance.
Uniform Probate Code
Capacity to make a Will 265-274; Insane Delusion 274-283

Wills: Capacity and Contests


3-5% of wills are contested on the grounds of capacity and undue influence
Mental Capacity: Testator must be capable of knowing and understanding in a general
way

1. The nature and extent of his property


2. The natural objects of his bounty
3. The disposition they are making of that property
4. Relating these elements to one another.
5. Will represent her wishes.
** Capability not knowledge **
Fraud occurs where someone intentionally misrepresents something to the testator,
with the intent of influencing the testators testamentary scheme, and the
misrepresentation causes the testator to dispose of his or her property in a way that he
or she would not have otherwise.
Fraud in inducement occurs when a person misrepresents a fact to the testator for
the purpose of inducing the testator to execute a will within certain provisions OR for the
purpose of inducing the testator to revoke a will.
KEY is that the misrepresentation does NOT go to the terms of the will per se but
rather concerns a fact that is important to the testator and may induce the testator
to dispose of his or her property differently in light of the misrepresentation
Fraud in the execution occurs when a person misrepresents the NATURE of the
document the testator signing. When either a person tricks another into signing a
document that purports to be the signers will but the signer does nto realize it or when
the testator realizes he or she is signing his or her will
Standing general rule is that a party will have standing to contest the validity of a will
or provision in a will only if that party will financially benefit if his or her challenge is
successful.

IN re Wrights Estate
Facts: Lorenzo Wright died at 29. The decedent left an estate consisting of two
parcels of land in Venice and an estate in salt lake city his formal home, and some

property to people but mostly to a friend named Charlotte. The notary, realtor and
two subscribing witnesses testified that they believed he wasnt of sound mind.
Didnt have extremely persuasive reasons
Holding: There is no evidence that testator suffered from settled insanity,
hallucinations or delusions. Testamentary capacity cannot be destroyed by showing
a few isolated acts. The opinions were trivial
Notes: Capacity to create a will requires less capacity than contract law.

Wilson v. Lane
Facts: Greegs property was divided equally to 17 beneficiaries, 16 of them
relatives. Katherine lane was not blood relative but cared for Greeg before her
death. Attorney testified that Greeg was mentally competent when choosing the
benefiaries The caveators challenged her capacity by saying she was eccentric,
aged. All that is required to sustain the will is proof that she was capable of forming
a certain rational desire in regards to her assets. A medical expert concluded she
was in a stage of dementia but this doesnt mean she cant form a will. She had a
guardian for her finances, but only for the concerns of her living alone from an
agency.
Holding: No testimony, expert or otherwise was offered to establish that she
suffered from a form of dementia sufficient to make her unable to form a will
Insane Delusion must show 1) testator labored under an insane delusion and 2) that
the will or some part of it was a product of the insane delusion moment of lucidity
In re Strittmaters Estate
Facts: An appeal admitting to probate the will of Louisa S. Appellants challenge on
her insanity. Medical witness was her physician her entire life. In her opinion she
suffered from paranoia and had a split personality. Her disease had been welldeveloped by the time she wrote her will. She left her whole estate to the womens
party
Holding: It was the product of her paranoid condition, especially her insane
delusions about the male, that led her to give her estate to the womens party. The
probate should be set aside.
Breeden v. Stone
Facts: Contested probate of a handwritten will. Mr. Breeden died in his home from
a self inflicted gunshot wound two days after being involved in a hit and run which
killed the other driver. Upon entering the home, the police found a handwritten
note which left everything to Sydney stone who attempted to probate it. Breedens
sister alleged a lack of capacity. The probate court entered the will the court found
cocaine and alcohol in his system, but the petitioners didnt prove by a
preponderance of the evidence that because of the use that he was not sound
mind when executing the will. Hanks test: one may have insane delusions for some
matters, but may still be able to transact business. Such insanity does not make
one incompetent to contract unless the subject of the contract is connected to the
insane delusion
Holding: The probate court correctly applied the tests he was of sound mind
when he executed the holographic will. Affirmed.
Capacity Burden of Proof

Majority Rule
o Due execution creates presumption of capacity
o Burden of persuasion on contestant
Accordingly, the will is admitted unless
Fact finder concludes that incapacity has been proved by a
preponderance of the evidence.
Minority Rule
o Due execution creates presumption of capacity
o Burden of production on the contestant
Accordingly, if and only if contestant comes forward with evidence
supporting a finding of incapacity
Then the will is not admitted unless the fact finder concludes that
capacity has been proved by a preponderance of the evidence.

Undue influence 283-305


Undue Influence protects against a wrongdoer taking unfair advantage of a
susceptible donor. Must show:
1. the donor was susceptible to undue influence
2. the alleged wrongdoer had the opportunity to exert undue influence
3. the alleged wrongdoer had a disposition to exert undue influence
4. there was a result appearing to be the effect of the undue influence
a. clear and convincing evidence .
Circumstantial Evidence
- susceptibility to undue influence
- opportunity to exert undue influence
- disposition to exert undue influence
- result appears to be the effect of undue influence.
Estate of Lakatosh
Facts: Roger Jacobs took advantage of the confidential relationship with Rose L to
siphon money. Spry, Guardian of her estate, filed for an accounting of a
constructive trust on Roger and then sought to revoke her last will on the grounds
of undue influence. Bench trial ordered that a constructive trust for $128k be
imposed on roger and her will be revoked. He appeals. Rose came to depend on
roger; gave him power of attorney, executed a new will and gave him all but $1k.
Atty Jacobs said she was not competent at the time to understand her
actions/estate. Rogers converted the money for his own benefit and for his friend.
Issue: Was there a confidential relationship? Did this result in receiving a bulk of
the state? Was there a weakened intellect at the time the will was executed?
Holding: Trial court affirmed Roger failed to carry his burden of proving the
absence of undue influences.
Presumption and burden shifting in undue influence cases
- In Lakatosh Roger had burden of proving that roses will was voluntarily made
because he had been in a confidential relationship with her and the suspicious
circumstances were present.
(1) Must have a confidential relationship- fiduciary, reliant, or dominant-subservient

(2) Suspicious Circumstances- may be satisfied by showing that the influencer procured the will
(3) If a presumption of undue influence is triggered, burden shifts to the proponent to rebut the
evidence.

Confidential relationship- some situations involving a trusting relationship the law


requires a person to be other regarding because of the potential abuse of trust
Suspicious circumstances In addition to a confidential relationships a contestant
must usually show the existence of suspicious circumstances.
Presumption and burden shifting
In re estate of Reid
Facts: Thomas Pluskat filed to set aside the attested will of Mary Lea Reid, the
adoption of Michael Cupit by Reid and the warranty deed executed conveying her
home and land to Cupit. Cupit a 24 year old male made an uninvited visit to her
home (for an interest in hold homes and his family had connections to the fact)
He attended law school and had a mother/son relationship with her. Evidence
said it was intimate. Cupit met with Atty Boutwell about becoming adopted so he
was her sole heir. Boutwell said it wasnt necessary then cupit asked him to
prepare a deed to convey her property to him. Cupit visited the farm and prepared
a will. The atty took every precaution to ensure she was competent.
Holding: Chancellor found there was undue influence attested will was the same
as the holographic/conflict of interest at the firm. Cupit alienated Reids family and
friends. Decision affirmed. Cupit argued Pluskat had no standing and was barred by
SOL this is correct but court said the case was just so unusual.
Lipper v. Weslow
Facts: Contest of the will of Mrs. Sophie Block on the ground of undue influence.
Ps are 3 grandchildren; Ds are here two surviving children (Frank son, also her
lawyer). The will did not leave anything to her grandchildren. Trial court set aside
the will. Ds appeal contending there is no evidence to support that the will was
procured by undue influence. She explains why the grandchildren werent left
money. However there was also evidence that Frank had issues with the deceased
son whose grandchildren were contesting the room. He lived next door to her,
potentially the will wasnt read and the explanation part of why no inheritance to
them was conflicted by evidence. Evidence was that she was of strong mind.
Holding: No evidence of undue influence. She was a sound mind Frank had the
opportunity to prevent her from receiving means of contract but no evidence that
he did (Will admitted she did receive some of the flowers).
Strategies if Contest is anticipated Record building (record discussions, exam of
capacity); maintain secrecy (inter vivos trust, gifts); sooth feelings (family meeting,
letters, etc.)
-

Bequests to lawyers and fiduciary appointments


o Undue influence many courts presume undue influence when an attorney
receives a gift
o Unethical conduct a lawyer cannot solicit a gift from a client, unless related
to the client
o Fiduciary appointments and conflicts of interest the lawyer should advice
the client concerning the nature of the lawyers interest in the appointment
and the availability of other counsel of the position.

Mistakes and Ambiguities 327-337; 343-351

Wills: Construction
Mistaken or Ambiguous Language in Wills
Plain meaning/no extrinsic evidence rule extrinsic evidence may be
admitted to resolve certain ambiguities but the plain meaning of the words of will
cannot be disturbed by evidence that the testator intended for another meaning.
(1) UPC 2-502- Will must be in writing; signed by testator; witnessed by two disinterested parties or
notarized. Holographic will is valid if signature and material portion of document is in testator's
handwriting. Extrinsic evidence allowed.
No reformation rulecourts may not reform a will to correct a mistaken term to
reflect what the testator intended the will to say.
o Anmheiter v. Arnheiterreformation not allowed; no extrinsic evidence
allowed.
o In re estate of Gibbs reformation not allowed, but can ignore certain details.
In this case, they picked the wrong persons address from the phone book
Mahoney v. Grainger
Facts: Sullivan executed a will and instructed her attorney to leave all of her
property to her 25 cousins equally. Her prior will left only two of the cousins. The
will was executed and the trial judge ruled that the term heirs at law only applied
to her maternal aunt and not to her 25 cousins. Certain cousins appealed this.
Issue: Can extrinsic evidence that a testator intended to dispose a property to
beneficiaries not named in the will be admitted when a beneficiary can be
ascertained from the face of the will?
Holding: NO. Heirs of law living refers to her aunt. The only heir at law Extrinsic
evidence would only be admissible to determine the meaning of the testamentary
language that is not clear in its application to the facts.
In re estate of Cole
Facts: The will of Ruth Cole states a bequest for Vining in the sum of two hundred
thousand dollars ($25,000). Appellant disputes the trial courts determination to
consider testimony of the wills scrivener. The scrivener said he used his computer
to copy and paste then changed the name to veta vining. And the number to 25K,
the amount chosen, but failed to change the words. Appellant offered no
contradictory evidence. Trial court found the testimony was reliable
Holding: Affirmed. Courts allow patent evidence from the face of the will.
Latent ambiguity the terms are applied to the facts are modernly allowed.
Notes:
o Patent Ambiguity evidence from the face of the will. In re Cole it is the
inconsistency between two hundred thousand dollars and $25,000.
Under traditional law, extrinsic evidence is not admissible to clarify a
patent ambiguity -- The court is confined to the four corners of the will
even if as a result the ambiguous devise fails and the property passes
by intestacy.
o Latent Ambiguity manifests itself only when the terms of the will are
applied to the facts.
Takes one of two forms

A description for two or more persons or things fit exactly


(equivocation)
A description for which no person or thing fits exactly but two or
more persons or things fit partially. (no exact fit)
o Personal usage

Openly Reforming Wills for Mistake


Erickson v. Erikson:
Held that extrinsic evidence of mistake by scrivener is admissible and if proved by
clear and convincing evidence, the court may reform the will (if an atty did this
intentionally, relief would be achieved from constructive trust)
In re Estate of Herceg
Facts: The will of Eugenia had no named beneficiary. Colomba Pastorino,
executrix, had petitioned for the construction of the will to be the same as the
decedents prior will which gave Sergio Pastorino (per sitrpes) then Colomba if he
had not been living. If the will was filed intestate, a niece and nephew would get
the estate. Gorman, the atty, filed an affidavit that some lines had been deleted.
The difficulty in this case is that the prior holding found that where a named
beneficiary had been deleted, it cannot be supplied by construction/reformation of
the will. Clear and convincing evidence standard. Look to evidence if there is a
mistake and then look for any probable intent to see if will meet the standard of
proof.
Holding: The testator intended the residuary beneficiary to be Colomba, and her
name should be inserted in the will. This was a small bequest which helped.

Deceased Beneficiaries 351-352, 357-361, 367-373; Changed Property 373-374,


379-384
Death of Beneficiary Before Death of Testator if the testators actual intent is not
evidence, the court will apply rules of construction that are meant to implement the
probable intent of the typical testator.
Lapsed and void Devises
Lapsed if a devisee does not survive the testator, the devise fails
- Nearly all states have enacted antilapse statutes that under certain circumstances
substitute another beneficiary for the predeceased devisee.
Specific and General Devise if a specific or general devise lapses, the devise falls
into the residue.
T's will gives her watch (specific) to A, $10k (general), and then the rest to C (residuary). A and
B predecease T. The watch and money then go to C.
Residuary Devise If a residuary devise lapses, the heirs of the testator take by
intestacy. If only a share of the residue lapses, such as when one of two residuary
devisees predeceases the testator., at common law then the lapsed share passes by
intestacy to the testators heirs rather than the remaining residuary devisees [no residue
of a residue]
Ex. After making several specific and general devises to various persons, T devises
the residue of her estate one-half to B and one half to C. B predeceases T. Bs onhalf share goes to Ts heirs by intestacy not to C.

Class Gift If a devisee is to a class of persons, and one member of the class
predeceases the testator, the surviving members of the class divide the gift
Ex. T devises $10,000 to the children of A (a class gift). One child of A, B,
predeceases T. At ts death, T is survived by another of another child of A, C.
Because this is a class gift, C takes the entire $10,000.
Void Devise If a devisee is already dead at the time the will is executed or the vise is
a dog or cat or some other ineligible taker, the devise is void, the same rules apply to the
lapse devise also apply to a void devise
Antilapse Statutes they do not prevents a lapse, RATHER they substitute other
beneficiaries, usually the dead beneficiarys descendants, if certain requirements are
met. A typical antilapse statute provides that if a predeceased devisee is related to the
testator, and is survived by descendants who survive testator, those descendants are
substitute for the predeceased devisee
1. Theory of presumed intent: must bear the particular relationship specified in the
statute
2. UPC 2-605: If a devisee who is a grandparent or a lineal decedent of
grandparent of the testator is dead at the time of execution of the will, then the
issue of the deceased devisee takes in place of the deceased.
a. 1990 UPC adds a devise to a stepchild
Presumed Intent the theory behind antilapse statutes is that for certain predeceasing
devisees, the testators would prefer a substitute gift to the devisees descendants rather
than for the gift to pass in accordance with the common law of lapse.
Scope an antilapse statute applies to a lapsed devise only if the devisee bears the
particular relationship to the testator specified in the statute.
T devises home to niece, B and residue to A. B predeceases T, leaving a child, C, who
survives T. What happens to As Share?

Anti Lapse as Default Rules


DEFAULT RULES because antilapse statutes are designed to implement presumed
intent, they are default rules that yield to an expression of the testators actual intent
that is contrary to the statute.
Ex. T devises her entire estate one-half to my son A and one half to my daughter
B, but if A or B or both do not survive me then I give such predeceasing childs
share to my friend F. If b predeceases T, leaving a child, C. At Ts death Ts estate
will pass one half to A and one half to F. The antilapse statute does not apply to bs
share, because T has provided expressly for the possibility of B predeceasing T.

Words of Survivorship
UPC 2-603(b)(3)
words of survivorship, such as in a devise to an individual if he survives me or in
a devise to my surviving children are not, in the absence of additional evidence, a
sufficient indication of an intent contrary to the application of this section
o ex. T devises Blackacre to my son Sidney, if he survives me. And vests the
residue of his estate to his wife , Wilma. Sidney dies in his fathers lifetime,
leaving a daughter, Debby. T is survived by Wilma and Debby. Who takes
Blackacre, Wilma or Debby? The issue is whether the words if he survives
me impose a condition of survivorship.
The majority of cases have held that an express requirement of
survivorship, such as if he survives me precludes the antilapse
statute.
UPC 2-603(b)(3) the term if he survives me is not enough to impose
a condition of survivorship and the antilapse statute applies
nonetheless substituting Debby for her father.
Class Gifts under lapse rules, a class gift is treated different from a gift to individuals.
If a class member predeceases the testators, the surviving members of the class divide
the total gift including the deceased members share UNLESS an antilapse statute
applies.
What is a class?

A class gift arises if the testator was group minded.


o Testator is group minded if he uses a class labels in describing the
beneficiaries such as to As children or to my nephews and nieces
o A gift names to beneficiaries who form a natural class may be deemed a
class gift if the court decides that the testator would have wanted the
survivors to divides the share of predeceasing beneficiary rather than for it
to lapse.
o Restatement 13.1
o Restatement of Property Wills and Other donative Transfers 13.2

Dawson v. Yucus
Facts: Stewart devised her interest in her late husbands farm house to two
nephews- Gene Burtle and Stewart Wilson. Burtle Died before Stewart (woman)
Can Wilson get the rest as a class gift as each was supposed to get ? Trial Court
ruled that it was NOT a class gift and entered extrinsic evidence to show that
Stewart wanted the property to go to her husbands side of the family.
Issue: Was this a class gift, or should the residues take the remaining ?
Holding: Affirmed Trial court the will did not create a class gift. The gift in the
clause to Burtle lapsed and therefore passed into the residue of her estate. There
was no language of the will that indicates she intended to create a class gift.
Notes:
Application of Antilapse Statutes to Class Gifts
Almost all states apply their antilapse statutes to a single generation class gift UPC
2-605
o Ex. Children, siblings
Ex. T, a widow, dies leaving a will devising Blackacre to my sisters and devising
her residuary estate to her stepson, S. When T executed the will, T had two sisters
living A and B. One sister died before will was executed leaving a child F who
survived T. A died during Ts lifetime leaving two children, D and E. T is survived
by B, D, E, F and S. Who takes Blackacre?
o Assuming antilapse statute applies to devise to sisters in most states B takes
1/3 share, D and E SPLIT 1/3 share and F takes 1/3 share.
o In minority of states, F does not share because C was dead when the will was
executed and black acre goes to B and split between D and E.
o If antilapse statute DID NOT apply to class gifts B, as the sole surviving
member of the class would take Blackacre

Summary Diagram

Changes in Property after Execution of Will


Ademption by Extinction
A will includes a specific devise of an item of property but the testator sells or
gives the item away before death. What happens to the devise?
o A specific devise of REAL or PERSONAL property is subject to ademption by
extinction
o The devise fails.

Ademption only applies to specific devises (to take away) you no longer get it.
o Ex. Gifts of my property at 123 main street my car my three carat
diamond ring given to me by my aunt jane
o DOES NOT APPLY to general, demonstrative or residuary devises
General testator intends to confer a general benefit and not give a
particular asset
Ex. A devise of $100,000 to A. If there is not $100,000 in case in
the testators estate at death, the legacy is not adeemed; other
property must be sold to satisfy As general legacy.
Demonstrative hybrid, a general devise yet payable from a
specific source
Ex. Testators will gives B the sum of $100,000 to be paid from
the proceeds of sale of my Apple stock If testator dies without
$100,000 in apple stock the devise is not adeemed and other
property must be sold in order to raise the full $100,000
Residuary Devise conveys portion of testators estate not otherwise
effectively devised by other parts of the will
Ex. A devise to A of all the rest, residue, and remainder of my
property and estate
o Identity theory (traditional rule) if a specifically devised item is not in
the testators estate, the gift is extinguished.
o Intent theory ( UPC) newer theory, if the specifically devised item is not
in testators estate, the beneficiary may nonetheless be entitled to the
replacement or cash value of the original item, if the beneficiary can show
that this is what the testator would have wanted.

Uniform Probate Code 2-606


Satisfaction of general pecuniary bequests
Satisfaction sometimes known was ademption by satisfaction may be applicable if
a testator makes an inter vivos transfer to a devisee after executing the will.
o If the testator is a parent of the beneficiary (or stands in loco parentis) and
sometime after executing the will transfers to the beneficiary property of a
similar nature to the devised by the will, there is a rebuttable presumption
that the gift is in satisfaction of the devise made by the will.
Exoneration of liens
If a will makes a specific disposition of property that is subject to a mortgage to
secure a note on which the testator is personally liable, it is presumed that the
testator wanted the debt to be paid out of the residuary estate
UPC 20697 a specific devise passes subject to any mortgage existing at date of
death, without right of exoneration regardless of general directive in will to pay
debts.
Abatement
Arises if an estate lacks sufficient assets to pay the decedents debts as well as all
the devises
UPC 3-902 if the testamentary plan would be defeated by the usual order of
abatement, the shares of the distributes abate as may be necessary to give effect
of the intent

Trusts385-400 Creation 400-417

Trusts: Characteristics and Creation


Basic Structure A transfers property to B for the benefit of C (and possibly others) --- one
party (settlor) transfers property to a second party (trustee) who holds and manages
the property for the benefit of one or more third parties (beneficiaries)

BASIC TRUST RULES

1. Same party can wear all 3 hats The same person can be settlor, trustee
and beneficiary at the same time as long as there is ANOTHER trustee or
ANOTERH Beneficiary
a. MERGER if the same party is both trustee and beneficiary and there is
NOOTHER trustee or beneficiary the legal title and the equitable title are
said to merge and the trust is terminated (bifurcation of the legal and
equitable titles is essential to a trust)
i. Bifurcation creates a difuciary duty between the trustee and
the beneficiaries. If the same person is both trustee and
beneficiary, one cannot hold oneself to a fiduciary duty so the
trust merges and terminates.
2. A trust will not fial for want of a trustee if the trustee decliens to serve,
dies or is unable to continue OR if the settlor forgot to name a trustee, a court
will appoint a successor trustee [once trustee accepts, the trustee can leave the
position ONLY with court approval or the consent of all the beneficiaries]
a. Where will creates a trust but fails to appoint a trustee the general rule is
to appoint the executor as trustee.
b. EXCEPTION if the court concludes that the powers given to the trustee
were personal, to be exercised by only THAT trustee the court will decline
to appoint a successor trustee and the trust will fail.
3. A trust is not created Until it is ffunded.
4. Co trustees must agree on action
a. Uniform trust code rejects the common law rule and prmits action based
on the vote of a majority of the co trustes.
A trust is a legal arrangement created by a settlor in which a trustee holds property as
a fiduciary for one or more beneficiaries .
Trustee takes legal title to the trust property, which allows the trustee to deal
with third parties as owner of the property.
Beneficiaries have equitable title to the trust property, which allows them to
hold the trustee accountable for breach of the trustees fiduciary duties.
o Typically entitled to periodic distributions from the trust income and
sometimes from the trust principal as well.
Types of Trust
Testamentary created by a will and arising in probate
Inter vivos created during the settlors lifetime by declaration of trust or by
deed of trust often as a will substitute to avoid probate.
Resulting trust arises any time a trust fails in who or in part, the courts use it
to require the party holding the property to return the property to the settlor or the
settlors estate if the settlor is dead.
Constructive Trust used to prevent unjust enrichment
Vocabulary, Typology and Illustrative Uses
A person who creates a trusts is the settlor, grantor, or trustor.

Inter vivos created during the settlors life


Testamentary created by will
Inter Vivos
Creation
Declaration of trust or deed
of trust
Type of Transfer
Nonprobate
Revocability
Revocable or irrevocable

Testamentary
Will
Probate
Irrevocable

Revocable.
Ex. O declares herself trustee of certain property for the benefit of O for life and
then on Os death, to pay the principal to Os descendants. O retains he power to
revoke the trust. Unless O revokes the trust, on Os death her descendants will be
entitled to the remainder of the trust property independent of any probate
administration of Os estate.
Trust for incompetent persons.
Ex. Os son A is mentally or physically impaired and is unable to manage property.
O transfers property to X in trust to support A for life, remainder to As
descendants, and if A dies without descendants, to Os daughter B.
Discretionary Trusts.
Ex. T transfers property to X in trust. The trust instrument gives X discretion to pay
any amount of income or principal to a or for As benefit. Or, x might be given
discretion to pay trust income to any one or more of a class of persons, such as A
and her descendants and to distribute the trust property to As descendants at As
death.
Testamentary.
Ex. H devises property to X in trust to pay the income to W for her life and then on
her death to pay the principal to Hs children.

THIRD PARTY RIGHTS

Bifurcation of Ownership
The trustee holds legal title to the trust property but the beneficiaries have
beneficial ownership. The law requires the trustee to subordinate the interest to
those of the beneficiaries (duty of loyalty). Duty of prudence trustee must hold an
objective standard of care and must administer the trust suited to its purpose.
o Two categories of issues arise from splitting of legal and equitable ownership
Asset partitioning the effect on the rights of third parties with
respect to the trust property and the property of the trustee personally
Fiduciary administration the powers and duties of the trustee and
the corresponding rights of the beneficiaries with respect to the trust
property and against the trustee (fiduciary administration)
Four Functions of Trusteeship
Custodial involves taking custody of the trust property and properly
safeguarding it.
Administrative includes accounting and recordkeeping as well as making tax
and other required filings
Investment involves reviewing the trust assets and making and implementing
an investment program for those assets as part of an overall strategy reasonably
suited to the purpose of the trust and the needs of the beneficiaries
Distribution involves making disbursements of income or principal to the
beneficiaries in accordance with the terms of the trust.
Trust v. Legal Life Estate
LEGAL LIFE ESTATE
Legal life tenant has no power to sell a fee
simple unless such a power is granted in
the instrument creating the life estate

EQUITABLE LIFE ESTATE trust


All difficulties of legal life estate are
resolved or mitigated by using a trust.

MODIFICATION & TERMINATION


Trust naturally end when all the trust principal is disburded pursuant to the terms
of the trust. Under special circustances however the terms of the trust may be
modified or the trust may be terminated prematurely
At common law, courts order the terms of a trust to be modified if
o 1. All the beneficiaries consent
o 2. An unforeseen change in circumstances materially frustrates the settlors
intent. The trust is modified to promote the settlors presumed intent under
the circumstnaces
** as general rule it is against public policy to use a trust to try to cheild ones assets
from ones creditor.
** traditional rule is that a will cannot revoke an intervivos trust unless the trust
expressly authorizes it.
Creation of a Trust
Creation of a trust requires:
o 1. Intent
o 2. Ascertainable beneficiaries who can enforce the trust
o 3. Specific property, the res, to be held in trust
o ** If testamentary OR to hold land** 4. A writing may be required to satisfy
the Wills act or the Statute of Frauds
Intent to Create a Trust
No particular form of words is necessary to manifest an intent to create a trust.
Do not even have to include trust or trustee is required.
Settlor need only manifest an intent to create a transfer of property to X for the
use and benefit of A is typically held to create a trust.
Testamentary trust is created by will. In a well drafted will the testators intent to
create a trust is stated clearly
Ex. All the residue of my estate, wherever situated, including lapsed devises, but
excluding any property over which I may have power of appointment, I get to XYZ
TRUST COMPANY, of Chicago, IL, as trustee to be held in trust and disposed of as
follows.
If intent is not stated clearly, it must be inferred.
Deed of Trust
Unlike a testamentary trust or an inter vivos trust of land, there are no particular
formalities required to create an inter vivos trust of personal property.
o Testamentary trust must satisfy the wills act because it is made by a will
o Inter vivos trust of land must satisfy statute of frauds
o Inter vivos trust of personal property no specific formalities required.
Jimenez v. Lee
Facts: Brought by P against her father to compel him to account for assets which
she alleges were held in a trust. Two gifts: Grandmother purchased a $1K bond for
education; second gift from one of Ds clients for $500. D cashed the bond and
invested the proceeds in a stock of a bank, entitled ownership as Jason Lee,
custodian for BEsty Lee. Did the same for the 2nd gift. Grial court found it was not
a trust he was just the custodian under the uniform gift to minors act. P claims

that because it was for education, created a trust. Ds attempt to broaden his
power by investing violated his duty to administer the trust
Holding: A trust exists here father was subject to liability. If he wrongly disposed
of the property, she could recover. If he sold trust property and acquired other
property with the proceeds, she could enforce the trust through the new property.

Declaration of Trust
Under a declaration of trust, the settlor simply declares himself to be trustee of
certain property. The settlor may also be a beneficiary of the trust.
o Ex. O makes a written declaration of trust declaring herself trustee of
$100,000 held at First National Bank, to pay the income to herself for life and
then on her death the funds are to pass to A. Even though O is the settlor,
sole trustee and the sole income beneficiary, this is a valid trust.
To have a valid trust, the trustee must owe fiduciary duties to someone other than
herself. If o were the sole trustee and also the sole beneficiary, the equitable and
legal titles would merge leaving O with absolute legal title, as under UTC 402(a)(5)
Declaration of trust v. Outright gift
o Outright gift requires the donor to deliver the property to the donee
o Delivery can be constructive or symbolic rather than physical, but delivery of
some kind is required.
o Intention alone is not enough to perfect the gift.
o If donor manifests an intention to make a gift but fails to complete delivery,
the question may arise whether the manifestation can be recharacterized as
a declaration of trust.
Hebrew University Assn v. Nye
Facts: The P obtained a judgment that it is the rightful owner of Abraham Yahudas
Library, a distinguished Hebrew scholar. The library contained rare books. The
controversy is between two Hebreew charitable institutions. Prof. Yahuda and his
wife indicated they wanted a scholarship research center in memorial to then.
Ethel met with P; P threw a lunch in her honor and here she described the property
and announced it as a gift to P. Ethel also approved a newspaper release. She had
prepared to ship the library but it was not shipped before her death. The complaint
alleged that P was the owner and entitled to possession. The facts show Ehtel
intended to make the gift inter vivos, it just hadnt been delivered.
Procedural posture:
o 1. A gift which is imperfect for lack of delivery will not be turned into a
declaration for a trust as it was not delivered. Allow case to be remanded.
o ON APPEAL: P gave memo contain a list of the contents of the library.
Constructive delivery has been found to exist in some situations: delivery of
key to safe deposit box, pointing out hiding places where money is hidden,
informal memos.
o
Holding: A public announcement followed by an itemized form are sufficient to
substitute for a formal instrument purporting to pass title. The P is the legal and
equitable owner.
Trust Property
Under traditional law, a trust cannot exist without trust property (res)

o Res does not have to be land or a substantial sum of money may be a


penny or any other interest in any type of property.
NECESSARY a specifically identified interest in property.
Unthank v. Rippstein
Facts: CP Craft handwrote a promise to make monthly payment to the appellee,
Iva Rippstein, for the next five years if he lived that long. Later, Craft added an
amendment to the letter stating that he was binding his estate to the monthly
payments and struck out the phrase, provided that I live that long. The appellee
sought to have the letter declared as a voluntary trust.
Holding: A donor does not create a trust by promising to make monthly payments
in the future. You must have an identifiable res (specific property) in a trust.
Beneficiaries 417-426 Writing 427-434
Ascertainable Beneficiaries
Beneficiary Principle
A private trust must have one or more ascertainable beneficiaries to whom the
trustee owes a fiduciary duties and who can call the trustee to account. (follows
more fundamental principle that a private trust must be for the benefit of the
beneficiaries)
Beneficiaries DO NOT need to be ascertained when the trust is created. Only need
to be ascertainable.
Clark v. Campbell
Facts: An estate of personal property was to be bequeathed to friend Common
law says cannot be a valid bequest to indefinite person. The word friends unlike
relations (okay to use) has no accepted statuary limitations, has no precise
sense. Friendship is broad and varied. Desire of flexibility
Issue: Must this fail for the want of certainty of the beneficiaries?
Holding: Where a gift is impressed with a trust ineffectively declared an incapable
of taking effect because of the indefiniteness of the class, the done will hold the
property in trust for the next taker under the will or the next of kin by way of
resulting trust.
UTC 402(c) a power in a trustee to select a beneficiary from an indefinite class is
valid. If the power is not exercised within a reasonable time, the power fails and the
property subject to the power passes to persons who would have taken the property if
the power had not been conferred.
UTC 402 a trust is created only if the settlor has capacity to create a trust, indicates
the intention to create a trust, the trust has definite beneficiary OR is a charitable trust, a
trust for the care of an animal 407 or trust for a non charitable purpose 408
Pet and Other Non charitable purpose Trusts
Beneficiary principle is not absolute it is not applicable to a charitable trust
Trend in cases codified by the UTC has been toward allowing enforceable trusts for
pen animals 408 and certain other non charitable trusts 409 which under
traditional law would be invalid for want of an ascertainable beneficiary.
In re Searights Estate

Facts: George Searight died testate. The third item of his will provided $1,000 for
his dog to keep and take care of it. If the $1,000 wasnt used up, he would then
have the remainder distributed to 5 other people. The probate court held the third
item as valid. In situations where the honorary trust is established for animals,
unless the instrument created such trusts limits the duration of the trust to human
lives, the court will create honorary trusts for animals with longevity. The amount
of time, including interest, is over 4 years and it is clear and the testator included a
time limit for the exercise of power this is less than he rule of perpetuities
Holding: The bequest for the dog does not violate the rule of perpetuities.
Affirmed.
Notes:
o Honorary Trust the transferee is not under legal obligation to carry out
the purpose. But if the transferee declines to do so, she holds the property
upon a resulting trust and the property will revert to the settlor or the
successors. Cannot violate RoP.
o Statutory Purpose for trust for pet, animal other non-charitable
purpose UTC 408
Honorary Trust
Statutory Purpose Trust
Transferee is not obligated to carry
- Statutory trust for pet animal or other
out settlors purpose
non charitable purpose
If transferee declines, she holds the
- Authorized by UTC 408-409 and
property on resulting trust and
UPC 2-907
property reverts to settlor or settlors
- Typically court may reduce if
successors
excessive and provide for
Used in Searights Estate
enforcement by settlor or court
appointee.

Written instrument?
The law of trusts, standing alone, does not require a writing to create a valid trust.
An oral intervivos trust of personal property, whether by declaration or by transfer
to another as trustee, is enforceable.
Oral Inter Vivos Trusts and Personal Property
UTC 407. Evidence of Oral Trust. A trust need not be evidenced by a trust instrument, but
the creation of an oral trust and its terms may be established only be clear and convincing evidence.
In Re Estate of Fournier
Facts: Fournier asked a couple who he was friends with if they could hold some
money for him. They said they would; he gave them two boxes, each with
$200,000 cash. He asked them to hold the cash in secret until his death, then give
it to his sister Faustina. Fournier explained to them that this sister needed it more
than the other sister. He requested secrecy but he told both Faustina and her
daughter that the money was being hold. The probate court found that Fournier
had an intent that the money pass through his estate, even though no instructions
that a trust had been created. The court ordered the money to go back as part of
his estate.
Holding: Clear error. There is no evidence that this is contradictory. Fournier
intended Faustina to take the money in her individual capacity. He created an oral
trust. Judgment vacated.

Notes:

Secret testamentary trusts and the wills act


Olliffe v. Wells
Facts: Ellen Donovan died and left her residuary estate to Rev. Wells to distribute
in such manner as his discretion which had been expressed to him. Ellens heirs
brought suit that the residue should go to them. Wells stated Ellen expressed to
him that her estate was to be used for charity. Document shows only evidence of
an outright fact
Issue: Can extrinsic evidence be admitted to show that there was the intent to
create a trust?
Holding: No. the extrinsic evidence would defeat the rights of the heirs at law. D
holds the property in a resulting trust for the testators heirs.
Notes:
o Semi-secret trust intent to create is clear, but the terms are unstated.
Extrinsic evidence not needed to prevent unjust enrichment of trustee. Trust
is invalid, not enforceable.
In the case of a semi-secret trust, the will makes a gift to a person in
trust, but does not name the beneficiary
o Secret trust devise is absolute on face (ex. Give to your sister, but then
tell your sister to give to certain others) extrinsic okay to prevent unjust
enrichment. Court can impose constructive trust.
In the case of a secret trust, the will makes a gift, absolute on its face,
to a named beneficiary. However, in reality, the gift was made in
reliance upon the beneficiarys promise to hold the gift property in
trust for another.
Semisecret Trusts
Secret Trust
- Intent to create trust appears on face
- devise is absolute on face
of will
- extrinsic evidence necessary to
- Terms are unstated
prevent unjust enrichment of
promisor/trustee
- Extrinsic evidence not needed to
- court will impose a constructive trust
prevent unjust enrichment of trustee
on trustee/promisor
- Trust is invalid, not enforceable
Will substitutes, intro 435-440 ; revocable trust 440-441; 449-452; 459-476

Nonprobate Transfers and Planning for Incapacity


Five Major Will Substitutes:
Revocable inter vivos trust
Life insurance
Various types of pay on death bank accounts
Transfer on death securities accounts
Pension accounts.
Legal questions raised by will substitutes
- Should Wills Act Formalities be required for validity?
o Wills Act NOT required for validity; low of wills do apply for substitutes (simul,
death etc.)
Revocable Trusts
Most flexible of all will substitutes, most like Will

Settlor can draft its provisions precisely to her liking.


Settlor of a revocable trust remains free to amend or revoke the trust at any time
and for any reason
Creation
o Deed of Trust whereby the settlor transfers to the trustee the property to
be held in trust . On the settlors death, the trust property is then distributed
or held in further trust in accordance with the terms of the trust.
o Declaration of Trust settlor simply declares himself to be trustee of
certain property for his own benefit during his life, with the remainder to
pass at his death in accordance with the terms of his declaration. settlor
retains power to revoke the trust and as trustee controls the management of
the trust property.

Notes.
Moon court held that a beneficiary of a revocable trust has no legally enforceable
interest while the trust is revocable. The trustee is subject to the control of the settlor
and only the settlor may enforce the trustees fiduciary duties.
If settlor is also trustee: any action by the settlor trustee that diminishes the
interest of a beneficiary cannot be a breach of trust but rather is an implied
revocation
JP Morgan Chase Bank v. Longmeyer court held that the trustee of a revocable trust
owed an affirmative duty of disclosure to the beneficiary even while the settlor was alive.
Revoking or Amending a Revocable trust
UTC 602(a) An inter vivos trust is revocable unless declared to be irrevocable
To amend or revoke a trust, the settlor has to follow precisely the method for
amendment or revocation specified in the trust instrument.
Patterson v. Patterson
Facts: Before passing away, Darlene P executed an amendment to the trust. She
was removing her son, Ron, as a beneficiary . On summary judgment the court
invalidated the amendment. The trustee, Randy P, appeals. Darlene said in the
trust that she had intentionally not provided anything for this son since he was
already provided for. Ron won on summary judgment because the amendment
completely divested Ron of his interest
Holding: The terms of the trust do not provide a method for amending that makes
it exclusive uses for the word may Darlenes amendment is validthe terms of
the trust do not specify an exclusive method, so she was okay. She also did it with
clear and convincing evidence in writing.
Application of Subsidiary Law of Wills
Although a will substitute need not be executed in compliance with the statutory
formalities required for a will, such an arrangement is, to the extent appropriate,
subject to substantive restrictions on testation and to rules of construction and
other rules applicable to testamentary dispositions.
Will substitutes are still subject to substantive restrictions and testation (no undue
influence)
State Street Bank & Trust Co. v. Reiser

Facts: The bank seeks to reach the assets of an inter vivos trust in order to pay a
debt owned by the estate. Holds the bank can do so. Wilfred Dunnebier created an
inter vivos trust with the power to amend/revoke. He conveyed to the trust the
stock of 5 corporations. He also created a will which left his residuary estate to the
trust. 13 months later, he applied for $75k loan and 4 months after receiving it he
died in an accident and didnt have any funds to pay it.
Holding: Where a person places property in trust and reserves the right to amend
and revoke, the creditors may, following the death, reach in to satisfy the debts to
them, To the extent not satisfied by the estate, those assets owned by a trust
would have enabled the settlor to use the trust assets for his own benefit. Court
cites to IRS code and 2nd restatement.

Clymer v. Mayo
Facts: Mayo, a prof, and executed a will naming her husband as the primary
beneficiary. Clara unified the disposition of all her property through her revocable
trust (will, life insurance, etc.) They divorced, and she changed the beneficiary of
her insurance to Marianne LeFrance, but left the trustees as the beneficiaries of her
pension plans and James the life beneficiary of the trust.
Issue: Was James interest in the trust revoked because of the divorce?
Holding: Legislative intent a divorced spouse should not take under a revocable
trust in these circumstances. In the absence of an expressed contrary intent the
statute implies an intent on the part of the testator to revoke will provisions
favoring a former spouse (pour over trusts, but applies with trusts too)
Notes:
o Revocation on Divorce and Revocable Trust?
UPC 2-804 applies to both wills and will substitutes. Revokes a
disposition in favor of a former spouse in a governing instrument
which is defined in 1-201(18) to mean deed, will, trust , insurance or
annuity policy, account with a pay on death or transfer on death
designation, pension plan or other such nonprobate transfer.
o Ademption and Abatement courts have applies most of the rules of
construction from the law of wills to revocable trusts.
o Capacity and Limitation Periods for Revocable trusts capacity required to
make a donative transfer is higher in most states for a lifetime transfer than
for a testamentary transfer.
Revocable Trusts in Contemporary Practice

The pour-over will a will that contains an express clause giving some or all of the
decedents probate property to the trustee of the decedents inter vivos trust to hold and
distribute pursuant to the terms of the trust. Typically the pour over clause is the
residuary clause, but it need not be. The clause can transfer to the trust a special gift or
general gift but the norm is either the residuary clause or a general gift of money.
Standard Pour over will clause I give the rest, residue, and the remainder of my
estate to the trustee of my inter vivos trust, to hold and distribute pursuant to its
terms.
O sets up a revocable trust with himself or a third party of trustee. O then executes
a will devising his probate estate to the trustee of that trust.
o Ex. Provision. I give my residuary estate to the then acting trustee under
the trust agreement executed by me on ______, ______, 20__, and known as
the O 20___ Revocable Trust of which I am now trustee and X is named as
successor trustee to be added to the trust estate and held under that trust
agreement as in effect at my death
UTC 2-511 you are permitted to create a will and trust separately with pour over
provisions
UPC 2-804 applied to both wills substitutes. It revokes a disposition in favor of a
former spouse in an governing instrument
Lifetime Consequences
Property management by fiduciary
Planning for incapacity
Keeping title clear
No federal income, gift, or estate tax benefits
Probate Avoidance
Avoid ancillary probate
Continuity in property management
Privacy and more difficult to contest than a will
Not subject to ongoing court supervision
More leeway than a will in choice of law
Uncertainty about subsidiary law of wills
Probate nonclaim statute may not be applicable.
Life insurance
Shifts the economic risk of premature death to an insurance company.
Commonly used to insure against lost income on the death of a wage-earner. Term
life insurance is the most common covers a certain period of time. Whole life
insurance combines life insurance with a savings plan

Cook v. Equitable Life Aurance Society


Facts: Douglas purchased a whole life insurance policy naming wife, Doris, as the
beneficiary. They later divorced. The divorce decree made no provision about the
insurance policy. After the divorce, Douglas stopped paying the premiums on this
policy policy was then changed to paid up term policy. He then married Margaret,
and had a son, Daniel. Douglas made a holographic will which gave his insurance
policy to Margaret and Daniel. The will was admitted to probate and Doris was

awarded the money. M/D argue that strict compliance with policy provisions is not
required to change a beneficiary in all cases. Doris asserts the Indiana law that
attempts to change the beneficiary life insurance by will without more, is
ineffectual
Holding: It may appear that the court should be sympathetic to M/D but doing
that would cause the danger or eroding the law. Upheld in Doris favor. Douglas
slumbered on his rights
Elective Share 511-531

Limits on Freedom of Disposition: Protection of the Spouse


and Children
Protection of the surviving spouse
Separate Property States Whatever a spouse earns is his or hers, so the
protection against disinheritance is elective share
o Elective Share sometimes called forced or statutory share, under elective
share statutes, a surviving spouse can elect to take under the decedents will
OR to renounce the will and take a fractional share of the decedents estate
(typically 1/3 of all the decedents probate property PLUST certain non
probate transfers)
TRUE OR FALSE TRUE that the intestate share of a surviving spouse could,
depending ont he circumstances, be of a lower value than the surviving spouses
forced share.
o Forced share = elective share.
Separate Property
Community Property
- No automatic sharing of earnings;
- property earned or acquired during
whatever spouse earns or acquires is
marriage is community property
his or hers
- No elective share, because each
- Protection against disinheritance
spouse owns all earnings during
provided through elective share
marriage in equal, undivided shares.

Marital Property Systems


-

Partnership Theory
Elective share justified because
surviving spouse contributed to
decedents wealth
Surviving spouse should be entitled
to one-half of decedents property
acquired during marriage

Support Obligation
Older view that marriage entails a
support obligation
Support theory implies
o Smaller percentage applied to
all the decedents property
o A minimum amount
o Accounting for other resources
available for support of survivor

Same-Sex Couples a surviving cohabitating partner is not entitled to an elective


share unless married

UPC 2-12: the right of election may only be exercised by surviving spouse. If a rep
claims the elective share of an incompetent surviving spouse, the portion of the elective
share which exceeds what the survivor wouldve received is placed in a custodial trust.
Subsequently deceased surviving spouse IN most states under UPC 2-212 the
right of election may only be exercised by the surviving spouse or a representative of the
surviving spouse during the surviving spouses life
Wilson v. Wilson court held that a claim for an elective share filed by a rep on
behalf of incompetent surviving spouse was extinguished by the death for the
surviving spouse.
Incompetent surviving spouse
Abandonment In a minority of state the elective share is denied to a surviving spouse
who abandoned or refused to support the deceased spouse.
Nonprobate property
Sullivan v. Burkin
Facts: Mary Sullivan exercised her elective share. She seeks a determination that
assets held in an inter vivos trust should be considered as part of the estate.
Probate judge rejected claim. The trust established Ernest (husband) as the sole
trusteeincome was paid to him. On his death, the successor trustee was directed
to pay the income equally to the Ds (George and Harold Cronin). The husband left
in his will that he intentionally neglected to make a provision for his wife (they had
been separated for years)
Issue: Whether an intervivos trust with a remainder interest is an invalid
testamentary disposition if the settlor retained broad power to modify or revoke
the trust, receive income and invade principal during his lifetime?
Holding: Intervivos trust is not an invalid testamentary disposition because the
settlor retains broad power over the trust during his lifetime. A surviving spouse
has no right to an intervivos trust even if it is established to defeat the wifes
election. Trust was not testamentary in character inter vivos because had had the
power to modify.
Notes: Rule we now favor would treat as part of the estate of the deceased
assets of an intervivos trust created during the marriage by the deceased spouse
over which he or she alone had general power of appointment exercisable by a
deed or by will.
In re Estate of Myers
Facts: Whether a surviving spouses elective share includes pay-on-death assets?
The probate court found that Karen Myers assets (checking account, certificate of
deposit, and annuity) should be included in elective share of her spouse
Holding: Pay-on-death accounts/annuities are not included in elective shares.
Overruled Sieh- equated them to revocable trusts and to be included.

Trust administration: Intro/powers 579-588; Duty of loyalty 588-602

Trusts: Fiduciary Administration


The hallmark characteristic of common law trust is bifurcation

Bifurcation a trustee holds legal title to the trust property, and the beneficiaries
have equitable or beneficial ownership
o Benefits to separation of legal and beneficial ownership property
transferred in trust during life avoids probate at the settlors death. There is
no need to change title by probate administration upon the settlors death
because the trustee holds legal title.
Three Kinds of Trusts in Practice
Business trusts for commercial deals
o Common law or statutory trusts created for a commercial purpose such as
organizing a mutual fund or facilitating asset securitization.
o NOT donative in purpose, are integral to a commercial deal
o Involves exercise of freedom of contract NOT freedom of disposition.
Revocable trusts for nonprobate transfers
o Most common use of revocable trust today is a will substitute for conveying
property at death outside of probate.
o Modern law, revocable trust need not have property, at least not initially, if
it is to be funded by a pour-over will.
o Trustee of a revocable trust does not owe fiduciary duties to the
beneficiaries, but rather is subject to the control of the settlor for as long as
the trust remains revocable.
*** a revocable trust is little more than a nonprobate will that avoids
the burdens of probate in manner reminiscent of how trusts were once
used to defeat primogeniture and feudal incidents ****
Irrevocable trusts for ongoing fiduciary administration
o Administration of property by a trustee in accordance with the settlors
intent. Increasingly, the trustee of such a trust is a fee- paid professional.
Trustees Powers
Permitting the settlor to incorporate by reference in the trust instrument all or
some enumerated statutory powers
2. Changing the default law to give trustees a statutory list of powers, such as
under uniform trustees powers act.
The Duty of Loyalty most fundamental principle of the fiduciary obligation in trust
law. Trustee must administer the trust solely in the interest of the beneficiary.
Hartman v. Hartle
Facts: Dorothey Geick left 5 kids. Two son-in laws were executorsthey sold the
farm of the estate to one of her sons ho bought the property for his sister, who is
the wife of one of the executors for $3900. She sold the property to D for $5500.
Issue: Can trustee sell a property to his wife without permission by an order of the
court
Holding: No. A trustee cannot purchase from himself at his own sale, wife is
subject to the same rule. Property is owned by innocent purchases, cannot order a
reseale. Executors will be held to account for 1/5 of the profits made.
Notes:
In re Gleesons Will

Facts: Gleeson lease 160 acres of farm land, P, as trustee, leased a portion of the
real estate of the trust to himself as a partner of William Curtin and he received a
share of the profits from the farming operation on the real estate. Rule: Trustee
cannot deal in his individual capacity with the trust property.
Holding: P should have decide whether he chose to continue as a tenant or act as
trustee. His election to act as trustee meant he couldnt deal with himself.
Notes:
o The No Further Inquiry Rule If a trustee undertakes a transaction that
involves self dealing or a conflict between the trustees fiduciary capacity
and personal interest good faith and fairness are not enough to save the
trustee from liability
ONLY DEFENSE THAT A TRUSTEE MAY RAISE ARE THAT:
The settlor authorized the particular self-dealing or conflicted
action in the trust instrument
The beneficiary consented that after full disclosure
The trustee obtained judicial approval in advance
o *** Even if the trustee has such a defense, the beneficiary
remains entitled to judicial review of whether the trustee
acted in good faith and of the fairness of the transaction***
o Remedies for Breach Justifying
Compensatory damages
Beneficiary is entitled to disgorge the trustee of any profit made on the
transaction.

In re Rothko
Facts: Rothko left numerous paintings in his will to his children. Reis, Stamos and
Levine were the executors of his estate. Rothkos children filed suit claiming that
the executors entered into improper business transactions to sell the paintings
Issue: Whether executors fail to act unfairly in the transactions they entered into
on behalf of the estate? Whether an executor who acting prudently on the advice
of counsel may be livable for the coexcutors breach od trust ? Whether an executor
who is liable for making an improper transfer where he had duty to retain property
but chose to sell the property is liable for appreciation damages?
Holding: Yes. The executors not only held an interest that conflicted with the
interests of the estate, but they acted unfairly because their interests conflicted
with the interests of the estate. Executors may be held to the same standard as
trustees. Trustees may have an interest in a transaction with the estate but they
must not engage in the transaction unless they can show that they will not be
improperly influenced by those interest when dealing with the estate.
Duty of prudence 602-614; 643-646; Custodial/administrative functions 646649; Trustee Selection/Division 649-657
The Duty of Prudence
The distribution function Distribution function involves making disbursements of
income or principal to the beneficiaries in accordance with the terms of the trust, which
may be mandatory or discretionary
Mandatory trustee must make specified distributions to an identified beneficiary.

o If O transfers property to X in trust to distribute all the income quarterly to A,


X has no discretion over when, to whom or in what amounts to make a
distribution. It would be a breach of trust for X not to distribute all income
quarterly to A.
Discretionary trustee has discretion over when, to whom or in what amounts to
make a distribution. If o transfers property to X in trust to distribute all the income
to such of A, As spouse and As descendants in such amounts as the trustee
determines, X must distribute all income currently but has some discretion over to
whom and in what amounts.
Discretionary Distributions
Marsman v. Nasca
Facts: Sara M died, James Farr (her dad) was her lawyer. Trust provided
reasonable maintenance: of Cappy only paid $300 over 10 years, he experience
financial duty. Farr exempt from liability no intent to abuse
Holding: Yes, wills that give trustees power to pay principal have a duty to inquire
into the financial resources of the beneficiary.
Notes:

Extended Discretion
- Trustee discretion is
sole absolute or
uncontrolled
- In spite of extended
discretion, trustee is
still subject to judicial
review
- Trustee must not act
arbitrarily
or
capriciously or abuse
its
discretion,
and
must act in good faith.

Exculpatory Clause
- trustee is excused
from
liability
for
breach of trust
- If trustee is
draftsman, trustee
must show disclosure
of clause and its
meaning to settlor
- Cannot excuse liability
for bead faith,
reckless indifference
or intentional or willful
neglect

Mandatory Arbitration
- Claims for breach of
trust must be resolved
by arbitration
- Whether such a
clause is enforceable
is unsettled; authority
is scarce and
contradictory.

643
Permissive retention
Mandatory Retention and Deviation
Scholarly debate and legislative change
Revocable trusts and beneficiary authorization
The Custodial and Administrative Functions
Duty to Collect and Protect Trust Property
o Trustee must collect and protect property without unnecessary delay
Duty to Earmark Trust Property

o Trustee must designate property as trust property rather than the trustees
own
Duty not to mingle trust funds with the trustees own
o Trustee must not commingle trust funds with his own, even if trustee does
not use the trust funds for his own purposes
Duty to keep adequate records of administration
o Trustee must document important decisions and actions and the reason for
those decisions and actions
Duty to bring and defend claims
o Trustee must take reasonable steps to enforce and defend claims relating to
the trust.

Trustee selection and divided trusteeship


Choosing a trustee
o Settlor asks friend or relative to serve (individual trustee)
o Settlor names a bank or trust company (company trustee)
Delegation by a trustee
o Trustee may delegate the investment function but must exercise reasonable
care, skill and caution in selecting, instructing and monitoring the agent.
Division by settlor
o Co trustees
o Power of appointment
o Directed trust and trust protectors
o
Private trust company
o Lightly regulated private trust companies meant to serve as trustee of one or
more trusts within a single family
UPC 807 Delegation by Trustee (2000)
Notes
Duty to delegate
Duty of cost sensitivity
Liability of trustee and agent
Division by a settlor
(1)co-trustees
(2)power of appointment
(3)directed trusts and trust protectors

Trusts: Alienation and Modification


Alienability of Interest in Trust 687-717
Alienation of the beneficial interest
Discretionary trust The settlor intends for the enjoyment of the trust property
to be delayed enough to get the trust property at all times. Just to be safe you
probably want to put that in there. UTC 504
o Pure discretionary trust
Trustee has absolute discretion over distributions to the beneficiary

Creditor of a beneficiary has no recourse against beneficiarys interest


in trust
o Support Trust
Trustee required to make distributions as necessary for beneficiarys
need
Insulates the trust property from some but not all of the beneficiarys
creditors (child, spouses and suppliers of necessities)
o Discretionary Support Trust
Common for a trust to combine absolute discretion with a distribution
standard.
A spendthrift trust beneficiary of a spendthrift trust cannot voluntarily
alienate her interest in the trust. This is true even if the beneficiary is entitled to
mandatory distributions form the trust. Creditors cannot attach interest. Created
for the benefit of a person (often unable to control his own spending) that gives an
independent trustee full authority to make decisions as to how the trust may be
spent UTC 502.
o Created by imposing a disabling restraint on alienation of the beneficial
interest
Ex. O conveys property to X in trust to pay the income to or for the
benefit of A for life and on As death to distribute the property to As
then-surviving descendants. Per Stirpes
Trust instrument provides The interests of beneficiaries in
principal or income shall not be subject to the claims of any
creditor, or to legal process, and may mot be voluntarily or
involuntarily alienated or encumbered.
A cannot alienate and her creditors cannot attach her interest in the
trust. X is free to make distributions to or for the benefit of A
irrespective of any claims by a creditor of A

Scheffel v. Krueger
Facts: Mother, Lorie, filed suit asserting tort claims against the D. She alleged that
D sexually assaulted her minor child, recorded it, and put it on the internet.
Ordered him to pay $550k damages to satisfy the judgment, sought attachment
of the Ds interest in his irrevocable trust set up by his grandmother. Trial court
ruled the spendthrift provision bars from attachment. P argues legislation didnt
intend a shield from tort creditors especially where there was a criminal act. Public
policy, wont be able to use the trust anyway because hell be in jail.
Issue: Whether a trust purpose for support and maintenance may still be fulfilled
where the beneficiary faces a criminal sentence for sexual assault. Whether a
statute that bars creditors from claiming an interest in a beneficiarys trust makes
an exception for tort creditors
Holding: No exception for tort victims the purpose of support and maintenance
trust may still be fulfilled while the beneficiary is incarcerated and after he is
released. The statute that bars creditors from making a claim against a
beneficiarys trust interest does not make an exception for tort creditors. Where
the legislature was made specific exemptions, the law must presume that no other
exceptions were intended.
o Rather, by its plain language the statute applied where a trust's governing instrument provided a
beneficiary was not able to transfer his or her right to future payments of income and principal,

and a creditor of a beneficiary was not be able to subject the beneficiary's interest to the payment
of its claim.
Notes: Legislature excludes other scenarios from constituting an exception to a
rule by listing specific exceptions. The court may not go against the manifest intent
of the legislature just because the beneficiary is a criminal. Exceptions include for
those who provide services necessary: Lawyer, physicians, grocers.

A self settled asset protection trust UTC 505 a person cannot shield assets
from creditors by placing them in a trust for their own benefit. EVEN IF the trust is
discretionary, spendthrift or both, the settlors creditors can reach the maximum
amount that the trustee could under any circumstances pay to the settlor or apply
to the settlors benefit.
o Ex. O, a surgeon, transfers property to X in trust to pay so much of the
income and principal to O as X determines in Xs sole and absolute
discretion. The trust includes a spendthrift clause. 5 yrs later, O botches a
routine surgery causing grievous injury to the patient, A. A may enforce a
malpractice award of damages against the entire corpus of the trust because
X could, in Xs discretion, pay the entire corpus to O.
UPC trust code 504
Federal Trade Commission v. Affordable Media LLC
Facts: Defendants Denyse and Michael Anderson were the trustees of a trust in
the Cook Islands. The united states district court issued a temporary restraining
order to have the funds transferred to the united states for the purpose of a trial
where the defendants were charged with fraud.
Issue: Whether the party demonstrates categorically and in detail that he is
unable to comply with the repatriation section of a preliminary injuction to transfer
to the united states all assets under their control directly and indirectly because
the assets are in trust under a trustee that refuses to relinquish the proceeds?
Holding: No. Ds were protectors of the trust and could have forced the trustees to
turn over the proceeds. Furthermore, the defendants showed they were aware of
their ability to force the trustees to transfer the money. After they stated that they
could not comply with the order the commission revealed that the Ds were
protectors of the trust. Thereafter Ds attempted to resign as protectors of the trust.
Notes: United States courts will penalize trustees of trusts in foreign lands if the
trustees are domiciled or resident of the united states and do not comply with
orders in regards to the over seas funds.
Trusts for the State Supported
Self Settled Trusts
Modification and termination 717-736; Removal 736-737; Protection from the
unintentional omission 562-577
Modification and Termination
Settlor consents
o Settlor plus all beneficiaries may modify or terminate
Settlor does not consent

o Claflin Doctrine
Consent of all the beneficiaries and
Not contrary to a material purpose of the settlor
o Equitable Deviation Doctrine
Claflin v. Claflin a trust was established for the testators son, with principal to be paid
to the some at age 30. After age 21 the son sued to terminate the trust arguing that he
was the sole beneficiary. Invoking the same courts earlier decision upholding the
spendthrift trust the court refused to permit termination as it would violate the intent of
the testator.
In re Estate of Brown
Facts: Brown created a trust to be used for the education of the children of his
new phone, WOolson S. Brown. After the accomplishment of the education trust
purpose, Brown directed the income of the trust to be used for the care,
maintenance, and welfare of his nephew Woolson Brown and his wife Rosemary
brown so that they would be able to live in the style and manner to which they
were accustomed.
Issue: Whether a trust is a support trust where the trustee must distribute all the
remainder income to specified beneficiaries after the initial purpose of that rust
fulfilled? Whether the material purpose of a trust was fulfilled are the education of
the settlors nephews children
Holding: The trustee must pay the amount of the remainder of the trust income to Woolson and
Rosemary Brown as is as needed for them to live in the style and manner to which they are accustomed
for the remainder of their lives. The material purpose of a trust that provides for the education of a
beneficiarys children ,and then for the beneficiary and his wife to live in a lifestyle to which they were
accustomed, is not satisfied after the beneficiarys childs education is complete. The settlor did not
merely name successive beneficiaries, but expressed intent to provide for the lifelong income of the
beneficiaries. Therefore this the second purpose is material
Notes: The purpose of a trust will be inferred wherever the settlor has expressed
that a distribution be made for a specific purpose, more than just for specific
beneficiaries.

Reform of Claflin Doctrine


UTC 411
Restatement 65
- Preserves material purpose
- Weakens material purpose
o Authorizes termination of
- Weakens requirement of
reason outweighs material
beneficiaries unanimity
purpose
o Authorizes termination if
- Preserves requirement of
interests of absent
beneficiary will be
beneficiaries unanimity
adequately protected

Deviation Under UTC 412


(a) The court may modify the administrative or dispositive terms of a trust or
terminate the trust if, because of circumstances not anticipated by the settlor,
modification or termination will further the purposes of the trust. To the extent
practicable, the modification must be made in accordance with the settlors probable
intention.
(b) The court may modify the administrative terms of a trust if continuation of the
trust on its existing terms would be impracticable or wasteful or impair the trusts
administration.
(c) Upon termination of a trust under this section, the trustee shall distribute the trust
property in a manner consistent with the purposes of the trust.
Extension to dispositive provisions
In re Riddell
Facts: The trustees parents established trusts for the benefit of the trustee, his
wife and the settlors grandchildren. The trust provided that the grandchildren
would receive the benefits until the age of 35 when the trusts would terminate and
the trustee would distribute the principal to the grandchildren. The trustees
daughter suffered from schizophrenia; she was not expected to live independently
for the remainder of her life. The trustee sought to create a special needs trust
on his daughters behalf, instead of distributing the trust principal to her. The
appellate court determined that the trial court properly found that it possess the
power to modify the trust pursuant to the trust and dispute resolution act.
Issue:
Holding: Special needs trust was created by 3rd party (parents of special needs
person)
Notes: Equitable deviation allows the court to modify the administrative or
distributive provisions of a trust if circumstances not anticipated by the settlor
mean that modification or deviation will further the purpose of the trust.
Ladysmith Rescue Squad Inc v. Newlin
Facts: Cosby died 2004, his will created a charitable trust that named Newlin and
Howell as trustees. Trust named 4 people as beneficiaries who were to receive
income payment for their lifetime. Interests in trust were insulated from creditors
by a spendthrift provision and they were not permitted to withdraw form the trust
corpus. After the death of all 4 beneficiaries trust was to be divided between Fire
dept and Ladysmith. In 2009 2 beneficiaries still alive, Fire seeks court order to
divide trust into two trusts, ladysmith and fire dept , then terminate the fire depart
trust and pay out the income beneficiaries and fire dept immediately. Ladysmith
said that granting the motion would frustrate Cosbys intent.
Issue: May a court modify or terminate a trust if such actions will not further the
purpose of the trust?
Holding: No.
Notes: For wills and trusts the testators or settlors intent prevails over the
desires of the beneficiaries; intent is ascertained by the language of the testator or
settlor used in creating the will or trust.
o TAKE AWAYS:

The beneficiaries other than ladysmith want immediate gratification


this isnt an unanticipated circumstance and doesnt warrant
termination under the common law approach or the UTC.
SURPRISINGLY courts routinely allow termination when the trustee
requests it and all beneficiaries agree.

Trustee Removal
Typically viewed as a remedy for breach of trust.
Modern law, trustee removal is more freely granted, effectively as a modification of
the trust
Professionally drafted drugs commonly include a provision that overrides default
law of trustee removal by authorizing the beneficiaries to remove the trustee and
appoint as successor an independent corporate trustee.
o Power to remove and replace the trustee can also be retained by the settlor
or given to a third party.
UPC 706
562-577

Protection Against Unintentional Omission

Spouse Omitted from Premarital Will


UPC 2-301
Statutes that correct for testators assumed mistake in neglect to update a
premarital will by looking to intestacy for what a typical married person would want
to pass to a surviving spouse.
In re Estate of Prestie
Facts: Maries and W.R were married, divorced and he was diagnosed with disease.
He executed a CA pour over will, which gave entirely to heir son. They married
again. His sight worsened she took care of him. He amended trust to give her the
condo in vega. After he died she petitioned for intestate succession as their
remarriage revoked the will. Probate court found his will to be revoked for Marias
omission. Son argues she was being provided for by being given the condo
Holding: An amendment to an inter vivos trust (condo) cannot rebut the
revocation of a will upon remarriage to an unintentionally omitted spouse.
Affirmed.
o Under Nevada law where a will does not provide for a surviving spouse who
married the testator after his will was executed, the presumption of
revocation may only be rebutted by evidence of a marriage contract or a
provision of the will that either provides for the surviving spouse or indicates
an intention not to provide for the spouse
o
Notes: An amendment to an inter vivos trust in favor of a spouse is insufficient to
rebut the presumption that marriage revokes prior will; pour over will
Unintentional Disinheritance of a child
Permitted heir statutes designed to prevent the unintentional disinheritance of a
child

o Many permitted heir statutes apply only to children born AFTER the execution
of a will, but some protect children alive when the will was executed as well
as after born children.
UPC 2-302 If a testator fails to provide in his will for any of his children after the execution of the will,
the omitted child receives a share in the estate as follows: i) if the testator had no children- will receive a share
qual in value to that which they would have recovered if the testator died intestate, ii) if there were children
living- entitled to a share limited to devises made to the then-living children (may have to abate), iii) doesn't
apply if the omission was intentional or if the child was provided for in another way.
Gray v. Gray
Facts: Decedent made a will giving his estate to his spouse, omitting children from
prior marriage. He had another child with a spouse, and didnt give to son. Probate
court found son, Jack, was entitled to distribute from the pretermitted child statute
(wasnt alive when will executed)
Holding: State supreme court held that an omitted child is not entitled to share if
the testator had other children and devised all of his estate to the other parent
instead of the omitted children. Exception applied, Jack couldnt review.
Notes: A child born after the execution of a will may not claim a share of the
estate if the will left substantially all of the estate to the surviving parent and the
testator had another child when the will was drafted who was omitted form the will

o
Blanket Disinheritance
most states the permitted heir statue can be avoided by providing for descendants
with representation.
Courts have been strict in requiring the testator to indicate clearly an intention to
disinherit a child, either by express words or by necessary implication
o In re state of Robbins Except as otherwise expressly provided by this
will I intentionally make no provision for the benefit of any other heir of
mine. The court held the langue did not disinherit a natural and an adopted
child.
In re Estate of Jackson
Issue: Whether the assets of revocable inter vivos trust are subject to the
pretermitted heir statute?
Holding: Pretermitted heir statute does not extent to a revocable inter vivos trust.
Notes: The permitted heir statute applies only to wills and not to revocable trusts

Power of appointments 795-816

Trusts: Power of Appointment

A settlor may give someone other than the trustee a nonfiduciary power to
distribute trust property this power is known as power of appointment
o Build flexibility into an estate plan
o Commonly used for tax planning and asset protection
Power of Appointment gives the done the power to override the distributive
terms of the trust and to direct the trustee to distribute some or ALL of the trust
property outright to the appointees

Benefits of Power of Appointment


Changes in circumstances
o Postpone and delegate decisions about who will receive future distributions
of trust property
Tax avoidance
o Can be structured to avoid estate or gift tax when exercised
Asset Protection
o Can be structured to avoid claims by creditors of the power holders.
Terminology and relationships
Parties
Donors the party who creates the power of appointment
Donnee the party who holds and has the right to exercise the power of
Appointment.
Appointees the individuals to whom the property is actually appointed; the
individuals in whose favor the power is actually exercised.
Objects or permissible appointees the class of individuals to whom the
property may be appointed; the group of eligible appointees in whose favor the
power may be exercised
Takers in default appointment or takers in default the individuals who are
identified in the instrument creating the power who are to take the property is the
donee fails to exercise the power
Appointive property the property that is subject to a power of appointment;
the property that the donee may appoint.
Creation
To create power of appointment:
o Donor must manifest the intent to do so, either expressly or by implication
Do not need to use the words power of appointment or appointee
No technical words are necessary , only intent to create a
discretionary power.
Precatory words [words that express a mere wish or desire] do not
constitute the intent to create a power absent additional evidence of
such an intent.
o Power of appointment confers discretion on the donee the donee may
chose to exercise the power or not.

Ex. Aunt executes a will in 2013 giving her tangible property to my


niece Wendy, to dispose of in accordance with a letter addressed to
Wendy dated Jan. 4th 2012, which is in my safe deposit box Wendy is a
trustee who has fiduciary duty to follow Aunts instruction. Wendy
does NOT have a power of appointment
General and Nongeneral powers
General Power one that may be exercised in favor of the donee, donnes
estate, creditors of the donee or creditors of the donees estate.
o Ex T devises property to X in trust to distribute the income and principal to
such of As creditors as A shall appoint by deed.
Non-general power (aka Special power) one that the donee can exercise in
favor of anyone except the donee, the donees estate, creditors of the donee, or
creditors of the donees estate.
o Ex. T devises property to X in trust to pay the income and principal to any
person whom B appoints by will except that B may not appoint to herself, her
estate, her creditors or the creditors of her estate
Time and Manner of Exercise
Inter vivos power (or lifetime power) must be exercised, if at all, by a writing
or deed executed by the donee inter vivos
Testamentary power one that must be exercised, if at all, by the donee at
death, typically in his or her will.
MANNER OF EXERCISE
TIME OF EXERCISE
Deed (lifetime power)
During life
Will (testamentary power
At death
Deed or will
During life or at death

To exercise Power of Appointment


5. a donee must manifest an intent to exercise the power
6. the manner of expression must satisfy any formal requirements imposed by the
donor
7. the appointment must be a permissible exercise of the power
Estate Tax considerations
General Power
o Donee is treated as owner of appointive property and taxed accordingly
Nongeneral Power
o Donee is not treated as owner of appointive property for tax purposes
Exceptions
o A power subject to an ascertainable standard is treated in a non general
power for tax purposes
o A lapse of power is not taxed to the extent of $5,000 or 5% of the trust
corpus five-or-five
Flexibility without estate tax liability
Creditor rights
General rule is that a creditor may reach a debtors property, if necessary to satisfy
a debt. A power of appointment is generally considered a personal right and NOT
aproperty interest. Creditors cannot reach the power.
Creditors of a donee MAY however be able to reach the appointive property if
the power is exercised.

o Creditors right & Nongeneral power (special power) Creditors of a


donee of a s[ecial power of appointment have no right to reach the
appointive property, either before it is appointed or after it has been
appointed.
Donee is merely an agent for the donor with the power to appoint the
property for the benefit of others.
o Creditors right and general powerMODERN STATUTORY TREND
creditors of a donee of a general inter vivos power of appointment can reach
the appointive property even absent an exercise of the power by the donee.
RATIONALE is that holding a general power of appointment is
tantamount to ownership over the assets thereby subjecting them to
the creditors claim.
Irwin Union Bank & trust Co. v. Long
Facts: As result of a divorce judgment Philip Long owed Victoria Long $15k. V sued
the trustee of a trust in which P had an interest in an attempt to satisfy her
judgment. The trust granted P the right to withdraw up to 4% of the principal per
year. P had never exercised the right to withdraw principal.
Issue: May a creditor reach assets that are subject to a general power of
appointment if the holder of the power of appointment has never exercised his
right to reach those assets? No.
Holding: Court ruled that the right to withdraw was a general power of
appointment over 4% of the trust res per year. Court applied traditional view that a
donee of general power of appointment had no property interest in the appointive
property unless and until the power is exercised. Because P did not exercise the
power, Victoria had no right to reach any of the property
Notes:
Exercise of a power of appointment
To exercise a power of appointment:
(1) the donee must manifest an intent to exercise the power;
(2) the manner of expression must satisfy any formal requirements imposed by the
donor; and
(3) the appointment must be a permissible exercise of the power
Whether or not the donee has manifested an intent to exercise a power of appointment
is a question of construction.
Restatement (Third) of Property: Wills and Other Donative Transfers,
19.1 (2011)

Manifestation of intent
Beals v. State Street Bank & Trust Co.
Facts: Dexter was the beneficiary of a trust created by her father where she
received a certain portion of the income during her lifetime. Dexter had the power
to pay and dispose of the property as she may direct and appoint by her last will
and testament in favor of persons who would be entitled to such estate under the
laws then governing the distribution of intestate estates. During her lifetime,

Dexter requested that the trustees make principal payments by transferring almost
all of her share to her husbands family. In her will Dexter disposed her property to
the issue of her sister Margaret who predeceased her. At death Dexter owned some
assets outright while $88,000 was still in trust.
Issue: Whether a testator exercises a power of appointment over a trust in her will
if she does not express nor implies intent to excise that power?
Holding: Yes. The testator exercised her power of appointment even though she
did not express a specific intent to exercise the power because the power was
general. The holder of a general power of appointment is not expected to
distinguish between the property subject to the power and her own property. The
testators power was general because she treated the trust property as her own
during her lifetime. She had the use and enjoyment of the appointive property that
was initially places in her trust share. Also she relinquished the right to add the
trust property to her estate and gave way part of her power.
Notes: Where a beneficiary of a trust is deemed to have been given a general
power of appointment , they are not required to distinguish between the trust
property and their own property in to order to exercise the power.

Residuary Clause courts are split over whether a standard residuary clause in the
donees will that does not make any reference to power of appointment exercises a
testamentary power of appointment.
-

Majority Rules
Residuary clause does not
presumptively exercise a general or
non general power of appointment
Variation on whether contrary intent
may be shown with extrinsic
evidence or only by reference of face
of the will

Minority Rules
residuary clause exercises a general
power of appointment unless a
contrary intent affirmatively appears
a few jurisdictions also apply
presumption to non general power of
appointment if the residuary
devisees are objects of the power.

Restatement (Third) of Property 19.4 (2011):


A residuary clause in the donees will or revocable trust does not manifest an intent to
exercise any of the donees power(s) of appointment, unless the power in question is a
general power and the donor did not provide for takers in default or the gift-in-default
clause is ineffective.
Formal Requirements Imposed by the Donor
- Even if a donee manifests an intent to exercise a power of appointment, the
manner of expression must satisfy any formal requirements of exercise imposed by
the donor.
o Two issues arise under this requirement
The nature of the instrument required for exercise
Whether the donee must make a specific reference to the power.
Specific reference requirements
UPC 2-704 If a governing instrument creating a power of appointment expressly
requires that the power be exercised by a reference, an express reference, or a specific
reference, to the power or its source, it is presumed that the donors intention, in

requiring that the donee exercise the power by making reference to the particular power
or to the creating instrument, was to prevent an inadvertent exercise of the power.
816-832
Permissible exercise of the power
Appointment to an object impermissible exercise of a power. One that purports to
benefit someone who is NOT an object of the power, such an appointment is invalid.
Timmons v. Ingrahm
Holding: because settlors intent is the main concern, when a settlor employs a
technical term, that term should be used in accordance with its legal definition
unless the settlor obviously uses the term in a different sense. Here lineal
descendant legally refers to a person in ones descending line and includes
adopted children . Even though the settlor expanded the definition of children to
include his step children the fact that he didnt expressly expand the definition of
lineal descendants demonstrates that he intended to have lineal descendants
interpreted according to legal definition.
Notes: In determining the intent of the settlor, a technical term in a trust
instrument will be accorded its legal definition, unless it is obvious that the settlor
used the term in a different sense. Ordinary legal meaning.
Appointment in Further Trust
Brown v. Miller
Holding: A transfer made to a revocable trust controlled by one individual
constitutes a transfer made to the individual himself. The transfer to the bill miller
trust was appropriate under the terms of the trust language. The fact that the
transfer was made to bill miller trust rather than to bill himself does not invalidate
the transfer.
o Bill miller trust is a revocable trust entirely controlled by Bill. Bill had the
power to revoke the trust at any time to gain ownership of the trust
property .
o Since bill had the right to withdraw all the trust property he did not act in bad
faith.
Notes: Transfers of assets to a revocable living trusts are the same as transfers of
assets outright to the person owning the trust. -- a transfer made to a revocable
trust controlled by one individual himself.

o
Exclusive and nonexclusive powers
Whether a power is exclusive or non exclusive depends on the intention of the
donor as revealed by the governing instrument
exclusive the donee can appoint all the property to one or more objects
excluding the other objects
o to any to such of
non exclusivethe donee must appoint some amount to each object
o to all and every one; to each and every one
Salvage doctrines: Allocation and capture when a donee intends to exercise
power of appointment, but the exercise is ineffective for some reason, it may be possible
to carry out the donees intent through the doctrines of allocation and capture
(1)Allocation if the donee of a special power of appointment expresses the intent
to exercise the power of appointments, but inappropriately attempts to mix the
appointive property with the donees own property in the distributive clause
(typically in a blended residuary clause) the doctrine of allocation unblends the
property to ensure that only eligible objects receive the appointive property
(2)Capture If a donee of general power of appointment (1) expresses the intent to
exercise the power of appointment and (2) blends the exercise with the
distributive provisions of his or her own will (typically in a blended residuary
clause) if any of the appointment gifts fails for any reason, the donee is held to
have appointed the failed gifts to him or herself (captured the appointive
properly and the failed appointive property is distributed as a part of the donees
general assets.
a. Applies only to general powers and only if an attempted exercise of such a
power is ineffective of incomplete
Disclaimer, Release and Contract
Failure to exercise a power of appointment
General Power
Non general power
Loring v. Marshall
Facts: Hovey created a testamentary trust granting powers of appointment over
the trust income to his nephews to exercise in favor of their wives under limited

circumstances. If her nephews failed to make such an appointment, the trust


property was to be transferred to the Boston museum of fine arts and other
charities. Hovery nephew Cabot Jackson Morse, left the remainder of his estate to
Anna Morse. Morse never exercised his right in regards to the trust principal to the
trust created by Hovey. When she passed, the trustees were uncertain how to
distribute the principal of the trust.
Issue: whether a gift may go to the remainderman of a trust where a donee fails
to exercise his power of appointment
Holding: No if a donee fails to exercise his power of appointment a gift must go to
any potential appointees, not the remainderman of the trust. The testator did not
intend that the property do to the charities but only to the descendants of his
nephew who was a donee under her will . The testator left her property to her
sister and brother and her nephews. She did not include any other persons. The
property must be transferred to one of her stated nephews and not to the heirs at
law. By the other terms used in her will, the testator knew how to refer to a
disposition in default of appointmne.t
Notes: The terms of the will showed the testator intended to leave property to her
nephews. None of the terms showed that she intended to include any of her heirs
of law who were not related to her nephews.

Another difference between a will and a trust is that a will passes through probate. That
means a court oversees the administration of the will and ensures the will is valid and the
property gets distributed the way the deceased wanted. A trust passes outside of probate,
so a court does not need to oversee the process, which can save time and money. Unlike
a will, which becomes part of the public record, a trust can remain private.

You might also like